[ /tv/ /rf/ /vg/ /a/ /b/ /u/ /bo/ /fur/ /to/ /dt/ /cp/ /oe/ /bg/ /ve/ /r/ /mad/ /d/ /mu/ /cr/ /di/ /sw/ /hr/ /wh/ /lor/ /s/ /hau/ /slow/ /gf/ /vn/ /w/ /ma/ /azu/ /wn/ ] [ Main | Settings | Bookmarks | Music Player ]

No.88754 Reply
File: paul_dirac.jpg
Jpg, 68.14 KB, 400×294 - Click the image to expand
edit Find source with google Find source with iqdb
paul_dirac.jpg
Продолжаем неспешное обсуждение калибровочных теорий! Или четвертый физика-тред.Предыдущие:
>>54970
>>16169
>>41
>> No.88755 Reply
File: listen_here.jpg
Jpg, 96.32 KB, 604×431 - Click the image to expand
edit Find source with google Find source with iqdb
listen_here.jpg
>>88629
> — Физике учат по десятитомнику Ландау?
Нет. Это локальный мем. Сам поймешь.
> — Это правда, что иностранные лаборатории очень интересуются студентами ФОПФа и вполне могут предложить работать за границей?
А почему бы сразу не поступить за бугор? И про работу - нет, а вот уехать учиться можно со многих кафедр физтеха.
> На ФРТК МФТИ на кафедре микропроцессорных технологий студенты с 3-го курса могут проходить практику в "Интел". Есть ли подобное на ФОПФе и как это осуществляется?
Практика на базе с 3 курса есть на всех кафедрах физтеха.

Не зацикливайся. Илитарность фопфа обусловлена только задротизмом местной популяции. Учиться можно на фупме, проблемах, квантах или фивте (ха!), - отличие от фопфа только в меньшей концентрации ботаников. Впрочем, курсу к 4 это уже никого не ебет.
>> No.88809 Reply
Я программист. Люблю математику; физику знаю постольку-поскольку, почти всё, что помнил со школы -- забыл. Моя цель -- понять квантовый компьютинг. Насколько я понимаю, для этого не обязательно разбираться во всех тонкостях квантовой физики (могу ошибаться (и вполне вероятно, что ошибаюсь)).

Анон, будь добр, накидай мне список prerequisites.
>> No.88831 Reply
>>88809
Привет, анон. Список пререквезитов таков:

Квантовая суперпозиция, постулаты квантовой механики (как минимум о процессе измерения), теорему о запрете клонирования бы знать тоже не помешало (и заодно научно-популярно о квантовой телепортации почитать).
Кубиты. Нормировка, сфера Блоха и базовые знания линейной алгебры (произведение матриц скалярное / тензорное; обратные, унитарные, эрмитовы матрицы), а потом смело изучай квантовые вентили и преобразование Фурье.

Потом сможешь в квантовые алгоритмы такие как Квантовое Преобразование Фурье (по сути вентиль Адамара для n кубитов), ну и изучить весь протокол квантовой телепортации. Потом только можно говорить о квантовом компьютинге как таковом.
>> No.88898 Reply
>>88831
> ну и изучить весь протокол квантовой телепортации
Да, забыл сказать: это уже если ты хочешь заглянуть в основы квантового компьютинга. Предполагается знать о квантовой сцеплённости.
>> No.89132 Reply
File: madskillz.png
Png, 6.54 KB, 456×315 - Click the image to expand
edit Find source with google Find source with iqdb
madskillz.png
>>88754
Суп анон. Недавно проходили электростатику, и у меня появился вопрос (см. пикрилейтед). Когда задал этот вопрос репетитору, он завис на минуту, а потом сказал что, наверное, заряд не может быть нечетным. Что скажешь?
>> No.89135 Reply
>>89132
Не понял вопроса.
> заряд не может быть нечетным
Может.
>> No.89138 Reply
File: tumblr_mdrsjyW4Ce1qik4mio1_500.jpg
Jpg, 79.37 KB, 500×701 - Click the image to expand
edit Find source with google Find source with iqdb
tumblr_mdrsjyW4Ce1qik4mio1_500.jpg
>>89135
Берем два металлических шарика и заряжаем один из них 3e. Потом сталкиваем их и отодвигаем обратно. По идее, заряд должен поделится между этими шариками равномерно, но только вот 3e не делится на два.
Прошу прощения если опять неправильно задал вопрос.
>> No.89152 Reply
>>89138
> только вот 3e не делится на два
Почему же? Будет полтора e.
>> No.89155 Reply
File: 53b287d2_star-wars-princess-leia-sexy-carrie-fishe.jpg
Jpg, 442.75 KB, 2500×1933 - Click the image to expand
edit Find source with google Find source with iqdb
53b287d2_star-wars-princess-leia-sexy-carrie-fishe.jpg
>>89152
e неделимо, демон. У меня так в учебнике напмюисано.
это стеб?
>> No.89157 Reply
>>89155
А, вон оно что.
Неделим элементарный электрический заряд, няша. Ты себе представляешь металлический шарик, из которого выпилен строго один электрон?
> По идее, заряд должен поделится между этими шариками равномерно
Это справедливо, если заряд шариков существенно больше элементарного.
>> No.89159 Reply
>>89155
>>89157
А столкновение электронов уже кое-чем другим описывается, начинается на квантовая и заканчивается на электродинамика.
>> No.89190 Reply
File: star-wars-princess-leia-organa-wallpaper.jpg
Jpg, 425.31 KB, 2100×1722 - Click the image to expand
edit Find source with google Find source with iqdb
star-wars-princess-leia-organa-wallpaper.jpg
>>89157
> Ты себе представляешь металлический шарик, из которого выпилен строго один электрон?
Ну так в теории же.
> Это справедливо, если заряд шариков существенно больше элементарного.
Тогда пусть будет какой нибудь 10001e. Заряд существенно больше, но все равно нечетный.
>> No.89192 Reply
>>89190
На одном шарике будет больше электронов, но приближенно заряд поделится поровну.
> Ну так в теории же.
Использовать теорию для описания того, что она не описывает, нельзя, это грубая ошибка. Каждая физическая теория имеет границы применимости.
>> No.89193 Reply
File: 13536830188483.gif
Gif, 1662.26 KB, 400×300 - Click the image to expand
edit Find source with google Find source with iqdb
13536830188483.gif
>>89192
Все понял анон, спасибо. А как думаешь, мне указать репетитору о его ошибке, или нет?
>> No.89195 Reply
File: maxwell.jpg
Jpg, 111.06 KB, 807×807 - Click the image to expand
edit Find source with google Find source with iqdb
maxwell.jpg
>>89193
Я уже минут десять думаю над твоим вопросом.

>>89192-кун воду мутит. Но в его словах есть здравое зерно. Суть в том, что действительно надо корректно поставить эксперимент. Это можно сделать, например, следующим образом:

Мы хотим взять два металлических шарика, но то там много свободных электронов над зоной проводимости (газ электронов, описываемый как плазма, которая свободно летает по металлу + у границы еще вылетает на радиус Дебая). Как избавиться от электронов? Это просто. Во-первых, охлаждаем шарик почти до абсолютного нуля. Почти все электроны оказываются под зоной проводимости, а если ширина зона велика, то будет момент, когда точно все электроны под зоной проводимости.В-вторых, надо взять решетку из 15 атомов(чем больше атомов, тем больше вероятность, что появятся свободные электроны). Ок. Теперь заряжаем один из шариков 3 электронами. Подносим ко второму шарику и ... пишем уравнение шредингера, численно его решаем и получаем ответ, который зависит от времени и геометрии системы. Как-то так. Может быть, что и все 3 электрона улетят на другой шарик.

Как-то так. Не знаю, понял ли ты, но я понял.
>> No.89196 Reply
>>89195
Не мутит. >>89193-кун описывает явления с помощью теории, точность которой недостаточна для описания явлений.

>>89193
Спроси у него, что будет, если привести в соприкосновение нечётное число шариков. Как поведёт себя чётный заряд в этом случае, лол?
>> No.89200 Reply
File: images.jpeg
Jpeg, 3.99 KB, 147×160 - Click the image to expand
edit Find source with google Find source with iqdb
images.jpeg
>>88754
А вопросы только по школьной физике можно задавать?
А то я вожусь с диаграммной техникой, не догоняю немного т_т
>> No.89201 Reply
>>89200
Можно, для этого кафедра и создана. Только физиков тут мало, и особенно мало тех, кто хорошо знает школьную физику.
>> No.89212 Reply
File: 1358083088603.png
Png, 1.17 KB, 300×20 - Click the image to expand
edit Find source with google Find source with iqdb
1358083088603.png
>>89201
ну, у меня-то как раз не по школьной физике ^_^

ну, ладно, я помолчу
>> No.89218 Reply
File: episode_4_pic_large_3.jpg
Jpg, 61.79 KB, 800×339 - Click the image to expand
edit Find source with google Find source with iqdb
episode_4_pic_large_3.jpg
Школьник в треде, все в танк

Репетитор сказал что он еще немного подумал над вопросом, и решил что 2e распределятся между двумя шариками, а третий будет прыгать от одного шарика на другой, пока их не разъединят.
>> No.89219 Reply
>>89218
> третий будет прыгать от одного шарика на другой
С истошными криками "Джеронимо", ага.
>> No.89224 Reply
>>89218
сделайте меня развидеть это.
>> No.89226 Reply
>>89218
ну, какбы сохранение энергии, бро - ему не хватит топлива туда-сюда путешествовать, об этом, кстати, какбы намекает назввние раздела, который ты изучаешь - электростатика.

репетитор твой - сам еще школьник
ему и батарейки в часы не стоит доверять вставлять
>> No.89617 Reply
Здравствуйте, физики. Поясните, пожалуйста, мне - неучу - строение атома. В школьных учебниках атом представлялся как слепленное из шарообразных протонов в нейтронов ядро и электроны, кружащие по орбиталям вокруг атома, словно миниатюрные планетки, движущиеся по орбите вокруг своей звезды.
Как я понимаю, все это вранье и неправда, верно? Нейтроны и протоны, ввиду огромной величины сильного взаимодействия "размазаны" в ядре, а орбитали - это наиболее вероятные траектории движения электронов. Электроны которые могут находиться как внутри ядра, так и за сотню метров от него, пускай и с мизерной вероятностью. Мое миропонимание верно? И если верно, то могут ли в химических реакциях участвовать внутренние невалентные электроны, которые по случайности оказались вне своей орбитали.
Что такое волновая функция не знаю.
>> No.89619 Reply
>>89617
Всё верно понимаешь. Внутренние могут участвовать в химических реакциях, но такие события случаются раз в несколько жизней этой вселенной, надо думать.
>> No.89624 Reply
>>89617
Бытие - всё сущее, обозначение существующего мира как целого. Онтология - раздел философии, осмысляющий бытие. Субстанция - онтологическая первооснова всех вещей и явлений. Материализм - онтологическая концепция, которая утверждает, что субстанцией является "материя". Материя определяется её составляющими: пространством, временем, движением. Пространство, время, движение - интуитивно понятные каждому человеку категории, данные нам в ощущениях. Материализм противоположен идеализму и особенно его радикальному течению - солипсизму.
Физика - материалистическая наука, изучающая материю непосредственно. Физика состоит из трёх частей: философской базы, математических моделей материальных образований и экспериментальных данных. Теории, изучаемые физикой, определяются перечислением её свойств: философское описание явлений, для которых строится теория; аксиомы, определяющие математическую модель; сопоставление математическим абстракциям чувственно воспринимаемых объектов; предсказания теории. Предсказания теории - это сопоставление математическим утверждениям, полученным с помощью математического вывода из математических аксиом теории, философских описаний чувственно воспринимаемой реальности. Таким образом, всякая изучаемая физикой теория, во-первых, имеет определённые философские границы применимости, а во-вторых, является фальсифицируемой по Попперу. Теория является фальсифицируемой, если существует возможность провести эксперимент, сопоставляющий реальному поведению материи поведение материи, предсказанное теорией. Физическая теория - это изучаемая физикой теория, все предсказания которой соответствуют поведению материи (то есть не существует грубого расхождения предсказаний теории с наблюдаемой реальностью).

Исторически физика делила материю на вещество и поле. Полем называлась непрерывная проницаемая материя, а веществом - дискретные непроницаемые материальные образования. Современная физика стоит на принципах эквивалентности вещества и поля. Вещество в понимании современной физики есть квантовые возбуждения полей.
Квант - мельчайшая неделимая частица. Нечто называется квантованным, если оно состоит из квантов. Квантованность - это обобщение философской теории неделимости Демокрита, атомизма. Так как название "атом" было занято, в обиход вошёл термин "квант". Примеры того, что квантуется:
   - Группа студентов. Квантом группы студентов является отдельный студент.
   - Пельмени в кастрюле. Квантом пельменей в кастрюле является пельмень.

Существует изучаемая физикой теория, называющаяся квантовой теорией поля. Она, между прочим, предполагает, что материя состоит из четырёх фундаментальных квантованных полей: электромагнитного, гравитационного, сильного и слабого. Квантом электромагнитного поля является фотон.
Квантовая теория поля опирается на результаты квантовой механики. Выделяются абстракции, называемые элементарными частицами. Эти абстракции определяются перечислением их свойств. Физическая форма не входит в определение элементарной частицы. Таким образом говорить, что
> слепленное из шарообразных протонов в нейтронов ядро
протоны и нейтроны шарообразны, нельзя.
Предполагается, что элементарные частицы делятся на фундаментальные и составные, то есть комбинации фундаментальных частиц. Электрон является фундаментальной частицей. Протон и нейтрон - составные частицы.

Элементарные частицы образуют то, что в классической физике называлось веществом, и то, что в классической физике называлось силами.

Нельзя говорить о том, что элементарные частицы реальны; поскольку реально лишь то, что мы непосредственно воспринимаем органами чувств. Однако теория, которая использует понятие элементарных частиц, является физической, и поэтому вполне допустимо мыслить элементарные частицы как реально существующие.
> так и за сотню метров от него, пускай и с мизерной вероятностью
Возникает противоречие между математической моделью и физическими границами применимости.
> Поясните, пожалуйста, мне - неучу - строение атома.
Философская категория элементарных частиц (я не определял элементарные частицы строго; у элементарных частиц довольно много характеристик) плюс несколько математических формул и сопоставление математическим термам философских объектов. Затем - построение атома. Атом - это абстракция, определяющаяся перечислением свойств атома в терминах классической и квантовой физик, плюс снова матан. Каждая часть - и матан, и предположения, - существенны. Определение атома довольно велико.
> Как я понимаю, все это вранье и неправда, верно?
Это грубое упрощение, появившееся потому, что люди просто не понимают, что физика в процессе получения предсказаний оперирует абстракциями, которым вовсе не обязательно соответствуют чувственно воспринимаемые предметы. Результат работы физической теории (предсказание) можно пронаблюдать, а физические абстракции типа материальной точки, силы или электрона - нельзя.
>> No.89628 Reply
File: Оптический-обман.jpg
Jpg, 59.25 KB, 500×500 - Click the image to expand
edit Find source with google Find source with iqdb
Оптический-обман.jpg
>>89624
> физика в процессе получения предсказаний оперирует абстракциями, которым вовсе не обязательно соответствуют чувственно воспринимаемые предметы
>> No.89640 Reply
>>89624
Ух, сколько чуши-то понаписал.
На самом деле, хоть физика и изучает непосредственно материю (значение у онтологической материи и физической вообще-то разное), она не является "материалистической" в смысле онтологического начала.
>> No.89644 Reply
>>89640
Собственно говоря, от существования/несуществования "материи" ничего для физики не изменится.
>> No.89646 Reply
>>89640
А как всё обстоит на самом деле, умняша?
>> No.89648 Reply
>>89640
> значение у онтологической материи и физической вообще-то разное
Нет, одно.
> непосредственно материю
Строго говоря, опосредованно.
>> No.89650 Reply
File: WH_Female-Inquisitor_HRF_IFS.jpg
Jpg, 58.46 KB, 480×686 - Click the image to expand
edit Find source with google Find source with iqdb
WH_Female-Inquisitor_HRF_IFS.jpg
>>89646
>>89648
Прочь из треда, еретики! Развели тут марксизм-ленинизм, понимаешь
>> No.89651 Reply
>>89650
Сам ничего не ответил, а тех, кто стоит на одной из точек зрения, прогоняешь. Ай, как нехорошо.
>> No.89654 Reply
>>89651
Я ответил же. Вот >>89619 А бла-бла-бла не нужно жеж. Впрочем, я не прогоняю, это больше ирония.
>> No.89655 Reply
>>89654
Бла-бла-бла нужно. Вся мякотка как раз в бла-бла-бла. А вот "раз в несколько жизней этой вселенной" не нужно, потому что безосновательно.
>> No.89656 Reply
>>89648
> Нет, одно.
Нет, не одно.
>> No.89657 Reply
>>89656
Докажи.
>> No.89677 Reply
>>89657
Сам докажи.
>> No.89687 Reply
>>89677
Фу ты ну ты, повторюша.
>> No.89712 Reply
Анон, выручай. До понедельника мне нужно решить задачу по физике (первокурсота же)
Плоский диск (монетка) вращается вокруг оси, проходящей через его центр перпендикулярно к его плоскости с угловой скоростью омега. Найти время, через которое он остановится, если считать, что масса, радиус и угловая скорость известна.

Время выразил, но через тормозящий момент, который мне неизвестен. Препод сказал, что нужно разбить весь диск на бесконечно малые части, у каждой из которых будет своя масса и радиус. Но так как ему уже лет 60-70, то его отвлекли и он, естественно, потерял мысль и отправил меня решать. Что делать дальше я не знаю.
Физику-еле-сдал-на-47-баллов-кун
>> No.89713 Reply
>>89712
А за счёт чего он должен остановиться? Трение? Но про трение ничего не сказано. Угловое ускорение тоже неизвестно.
>> No.89714 Reply
>>89713
Нагуглил вот такую задачку, из нее и выразил время %%http://fizportal.ru/tower-9%%
Однако нужно каким-то неведомым способом найти этот гребаный тормозящий момент. Монета же остановится, ибо ее будет прижимать mg сила тяжести. Следовательно, будет возникать сила трения... Анон, я совсем запутался.
с: ласково милым - доброчан же!
>> No.89715 Reply
>>89714
> Монета же остановится, ибо ее будет прижимать mg сила тяжести
Почему?
>> No.89770 Reply
>>89646
Ну дык, Физика не является частью течения материализма, которое считает за первооснову всего сущего материю в филосовском смысле.
>>89648
Погуглил бы хоть. В космологии, да и в физике вообще понятие материи не включает в себя пространство. Советую гуглить matter - переводится в русскоязычных публикациях также как "вещество".
>> No.89771 Reply
>>89770
> в физике вообще понятие материи не включает в себя пространство
Включает, десу. Со времён ОТО.
>> No.89772 Reply
>>89770
> Физика не является частью течения материализма
Почему ты так думаешь?
>> No.89773 Reply
>>89770
> в физике вообще понятие материи не включает в себя пространство
> Физика описывает материю как нечто, ... задающее свойства пространства и времени — представление, идущее от Лейбница и, в дальнейшем, нашедшее выражение в общей теории относительности Эйнштейна
ruwiki://Материя_(физика)
>> No.89778 Reply
>>89770
> Советую гуглить matter - переводится в русскоязычных публикациях также как "вещество".
В физике "вещество" и "материя" не взаимозаменяемы. Вещество - часть материи.
>> No.89787 Reply
>>89771
Нет, не включает. И именно ОТО не включает.
>>89772
Потому что физика не определяет, что было первоосновой всего. Так же и она не может принадлежать какому-то одному мировоззренческому течению.
>>89773
Ну давай по твоей ссылке пройдёмся.
> объективная реальность, содержимое пространства
> Физика описывает материю как нечто, существующее в пространстве и во времени
> либо как нечто, само задающее свойства пространства и времени
Свойства она задаёт, искривляет, да. Правильней будет сказать, что геометрические свойства пространства-времени (а точнее метрика) зависят от материи, содержащейся в ней.
>> No.89789 Reply
>>89787
> Свойства она задаёт, искривляет, да.
Это значит, что пространство-время и поля связаны друг с другом. Пространство-время и поля в совокупности образуют материю. Не существует пространства-времени, свободного от физических полей. Не существует физических полей, которые были бы внепространственными и вневременными.
> физика не может определять, что было создателем и первоосновой всего
Это вообще не входит в задачи физики. Физика избирает материю первоосновой и работает, исходя из этого. Метод вспомогательной гипотезы же.
> И именно ОТО не включает.
У тебя есть физическое образование?
>> No.89791 Reply
>>89789
> > И именно ОТО не включает.
> У тебя есть физическое образование?
У меня есть. ОТО действительно не включает пространство в материю. Материя (сразу вспоминаем энергию, массу и прочие наблюдаемые величины) - это, грубо говоря, главный герой в сюжете под названием "путешествие элементарных частиц по заданной метрике", если говорить про high-energy.
> Пространство-время и поля в совокупности образуют материю
Глупая глупость. Пространство-время - это геометрия задачки, никакого отношения к физическому смыслу она не имеет. Например, топологические заряды, да и вообще топологическая теория поля - это геометрия, математика, часть законов. Уж не знаю, как доходчиво это донести до гуманитария,но интересна не абстрактная хуйня из марксизма под названием "материя" и какая-то устаревшая уже как 100 лет философия, а что-то более физическо осмысленное, для чего можно написать лагранжиан. А бэкграунд, будь то пространство минковского или Анти-де-ситтер, - всего лишь бэкграунд.
>>89787 - и доброчую вот этого.
>> No.89797 Reply
File: Particle_overview-ru.svg.png
Png, 28.83 KB, 470×273 - Click the image to expand
edit Find source with google Find source with iqdb
Particle_overview-ru.svg.png
>>89778
Учитывая, что веществом называют фермионы, а в зарубежной терминологии материей обычно называют, внезапно, фермионы - да, по части перевода эти термины взаимозаменяемы (да и в инглише нет отдельного слова для вещества, разве-что substance, но это химический контекст).
В то время как поля - суть математическое описание фундаментальных взаимодействий, и не являются материей / веществом, так как речь идёт о переносчиках фундаментальных взаимодействий - калибровочных бозонах, а не фермионах.
В свою очередь доброчую >>89791
>> No.89807 Reply
File: Witten_na_barrikadah.jpg
Jpg, 33.75 KB, 480×640 - Click the image to expand
edit Find source with google Find source with iqdb
Witten_na_barrikadah.jpg
>>89791
Вот этот дело говорит.

мимоперекатывался
>> No.89810 Reply
>>89791
> У меня есть.
О, тебя-то нам и надо, няша!
>>89624
> Поясните, пожалуйста, мне - неучу - строение атома.
>> No.89829 Reply
>>89810
> О, тебя-то нам и надо, няша!
Во-первых, тут есть более компетентный >>89797-кун.
Здесь вот >>89624 много буков. Я грешен, не осилил, но вроде как очень понятно человек пишет.
> Поясните, пожалуйста, мне - неучу - строение атома
Только если какие-то конкретные вопросы. Ибо я сам "сначала не понимал, потом привык."
>>89617 - всё правильно понимаешь.
> Всё верно понимаешь. Внутренние могут участвовать в химических реакциях, но такие события случаются раз в несколько жизней этой вселенной, надо думать.
Надо думать, что да, лол. Лень же считать вероятность, но суть в том, что это очень редкие события, которые в рамках химического приближения вообще никакой роли не играют.
>> No.89901 Reply
>>89617
Ну вообще правильно, но есть один момент
> это наиболее вероятные траектории движения электронов
В квантовой механике принято считать, что электроны не движутся, а оказываются в одном из своих вероятных состояний в момент влияния на систему.
До тех пор они сразу во всех возможных точках (протранственно описывается непрерывной волновой функцией, ведь точек может быть бесконечное множество). Ну или по дискретным уровням (а-ля нулевой или первый уровень), тут уж можно описать вектором состояния (его состояние, например - суперпозиция сразу двух уровней).
Надеюсь, не перегрузил деталями.
>> No.89902 Reply
>>89901
Наоборот, недогрузил.
>> No.89905 Reply
>>89902
Хм, а про что еще рассказать? Задавайте свои ответы же.
>> No.90161 Reply
Подозреваю, что мне тут не рады.
Но смиренно попрошу помочь.
Если решу эту задачу в течении часа, закончится месяц страданий.

В общем, как-то так:
В ртути плавает металлический куб со стороной a=13 см и плотностью ρ=8.6 г/см3. Поверх ртути наливают воду вровень с верхней гранью куба. Какова высота h (в см) столба воды? Плотность воды r0=1000кг/м3, ртути r1=13600кг/м3.

Хотя бы намекните, через какой закон можно связать плотность двух жидкостей с размером куба.
>> No.90162 Reply
>>90161
> в течение 4 часов
fix
>> No.90163 Reply
Хотя, уже трёх.
>> No.90164 Reply
File: КПЦКР34.png
Png, 2.87 KB, 309×238 - Click the image to expand
edit Find source with google Find source with iqdb
КПЦКР34.png
Поскольку куб неподвижен:
P = ρ a^3 g
Fa = (???) g V
Неясно, что делать с плотностью жидкостей.
>> No.90165 Reply
О! Предположил, что до воды было P = Fa, и нашел Vв ртути.
Оставшаяся площадь целиком будет ведь покрыта водой, так ведь?
>> No.90166 Reply
>>90161
Ну смотри. Я могу ошибаться, особенно если неправильно понял условие, но все же.
На куб вниз действует: mg=gp(мет)a^3
Вверх: Fa(ртути)=p(ртути)gx*a^2
   Fa(воды)=p(воды)*g*(a-x)*a^2
Отсюда выражаешь x. Высота воды будет a-x.
>> No.90167 Reply
>>90166
Лол, умножением зафейлил разметку, ну да ладно.
А вот >>90165 тут не верно, F=PS -> F=Pa^2. А если ты приравнял давление силе Архимеда, то это прямо-таки точно ошибка.
>> No.90169 Reply
>>90166
Большое спасибо! Позже напишу, правильно или нет.
>> No.90171 Reply
>>90167
Надо ведь приравнять силу Архимеда к тяжести, верно?
>> No.90173 Reply
Получил x=a(p-p0)/(p-p) и правдоподобные ~8.8 см.
Теперь остаётся разобраться с последней оставшейся задачей:

Пуля массой m1=16г, летящая со скоростью V=253 м/с, пробивает навылет подвешенный пластмассовый шар, масса которого M=2 кг. После вылета из шара скорость пули уменьшилась в 4 разa. С какой скоростью V1 будет двигаться шар в первый момент после вылета из него пули? Временем взаимодействия шара и пули пренебречь.
>> No.90174 Reply
>>90173
> /(p-p1)
>> No.90175 Reply
Поразмыслю над этим
http://www.pppa.ru/metodika/phy/machine16.php
Вроде бы, то.
>> No.90176 Reply
>>90173
mV=mV/4+M*V1;
отсюда V1=0.75mV/M;
А вообще, надеюсь что тебе сейчас нужно физику сдать и забыть, потому что задачки даже для школы несложные.
>> No.90177 Reply
>>90176
На самом деле, сейчас сижу и понимаю, что всё довольно просто.
Плохо знаю физику, но нужную инфу найти легко.
Да и связать с задачей не слишком трудно.
Просто у меня тут, скажем так, не очень благоприятная атмосфера.
И да, спасибо же, доброанон!
>> No.90178 Reply
File: i-came-ghostbuster.jpg
Jpg, 23.75 KB, 679×482 - Click the image to expand
edit Find source with google Find source with iqdb
i-came-ghostbuster.jpg
> зачтено
ИТТ все ответившие получают годовой запас добра.

Кстати.
> V1=0.75mV/M
Внезапно не верно.
Не думаю, что мог ошибиться в рассчётах.
Впрочем, какая разница.
>> No.90761 Reply
>>88754
Подскажите, пожалуйста, литературу. Нужно повторить школьный курс физики за 2 недели максимум (с физикой не имел дела уже 5 лет) для поступления на заочку.
>> No.91709 Reply
Хотел поделиться с вами описательной идеей о времени. Это будет гипотеза и прошу исходить из этого. Время неразрывно связано с пространством. Не будет пространства и не будет времени. Если мы возьмём движение, то оно происходит во времени и пространстве. Если мы возьмём любое видоизменение материи, то оно всякий раз будет происходить во времени и пространстве. Из этого получается, что время напрямую зависит от пространства. Также стоит отметить, что скорее всего время это свойство пространства, а не отдельное свойство вселенной. Следовательно, стоит искать зависимость времени от пространства. Возможно, то обстоятельство что свет исходящий из давно умерших звёзд будет доказывать эту зависимость, так во времени относительно наблюдателя в определённой точки они (звёзды) ещё живы, то относительно своего местоположения и наблюдателя этого местоположения эти звёзды уже мертвы.
  
   Если разбить пространство на равномерные отрезки, то и время, относительно пространства будет разбито равномерно, это означает, что у времени, если следить за материальными объектами во времени, можно заметить некую закономерность — все они, допустим, движутся не быстрее, чем могут двигаться относительно той силы, которая их заставила двигаться, следовательно, есть некая постоянная, которую подставив под определённое уравнение всегда будет сходить со временем, которое он должен будет пройти при определённых условиях.

   Это постоянная скорее, как и гравитационная постоянная, всё-таки меняется в определённых условиях, потому что имеет зависимость от пространства. Скорее всего, чем шире вселенная, тем медленнее течёт время.
>> No.91710 Reply
>>91709
Готов выдержать любую критику и ответить на все вопросы.
>> No.91711 Reply
File: einstein-ablert-study-1940-sized.jpg
Jpg, 73.78 KB, 616×418 - Click the image to expand
edit Find source with google Find source with iqdb
einstein-ablert-study-1940-sized.jpg
>>91709
> Следовательно, стоит искать зависимость времени от пространства.
Тут кое-кто уже подсуетился.
>> No.91712 Reply
>>91709
> Если разбить пространство на равномерные отрезки, то и время, относительно пространства будет разбито равномерно
Это верно только при линейной зависимости. При квадратичной - уже нет.
>> No.91714 Reply
>>91711

вообще время много от чего зависит. Если лекция не интересная - то идёт очень долго и никак не кончается. А если интересная - наоборот время ускоряется и жалеешь что это она так быстро кончилась.
>> No.91745 Reply
>>91714
Это из субъективных ощущений наблюдателя же. Особенность работы мозга.
>> No.91747 Reply
>>91745
Почему? Какие ваши доказательства?
>> No.91748 Reply
File: images.jpeg
Jpeg, 15.10 KB, 186×271 - Click the image to expand
edit Find source with google Find source with iqdb
images.jpeg
>>91745
> > Особенность работы мозга.
Особенность работы организма. За то время, Вудди кучу работы сделает, а человеки ещё и не проснуться толком не успеют.
> > из субъективных ощущений
А что какие то другие бывают? Вы меряете по субьективным ощущениями часов же.
>> No.91808 Reply
File: 1318341789486.jpg
Jpg, 28.37 KB, 600×480 - Click the image to expand
edit Find source with google Find source with iqdb
1318341789486.jpg
>>91748
> субьективным ощущениями
> часов
Омич-наркоман, продолжай пожалуйста.
>> No.91817 Reply
>>91808

Часы это же субьект. А вы принимаете их за универсальную точку отсчёта.
>> No.91821 Reply
>>91817
Не принимаем. Но ощущений у них-таки нет.
>> No.93010 Reply
File: prism.jpg
Jpg, 14.43 KB, 640×400 - Click the image to expand
edit Find source with google Find source with iqdb
prism.jpg
Анон-физик, у меня есть к тебе детский вопрос по оптике.

Допустим, у нас есть призма. В призме имеет место быть внутреннее отражение - я правильно пишу, да? - я нарисовал это слева.

Вопрос вот в чем. Если мы светим (снаружи) в ту самую грань призмы, от которой отражается (изнутри) зеленый луч, пройдет ли он сквозь призму так, как я нарисовал справа?
На правом рисунке не показано преломление, я в курсе. Вопрос не в преломлении, а в том, попадет ли луч с одной стороны призмы на другую.

Говоря другими словами, возможна ли такая форма призмы, при которой лучи с двух сторон будут совмещаться таким образом, чтобы наблюдатель, находясь с одной стороны призмы, видел совмещенные картинки с двух других сторон? Примерно так, как это происходит с полупрозрачным зеркалом, только с минимальными потерями света.
>> No.93020 Reply
>>93010
Да, первая грань должна полностью отражать зеленый луч и пропускать коричневый. Вторая грань, где свет выходит, может быть обыкновенной стекло-воздух. Можно даже и оставшуюся грань сделать как первую, тогда будет совмещаться три изображения.
>> No.93053 Reply
>>93010
> Допустим, у нас есть призма. В призме имеет место быть внутреннее отражение -- я нарисовал это слева.
допустим свет отражается несколько по-другому, но сойдет
> Вопрос вот в чем. Если мы светим (снаружи) в ту самую грань призмы, от которой отражается (изнутри) зеленый луч, пройдет ли он сквозь призму так, как я нарисовал справа?
свет, свет проходя через призму, преломляется в любом случае, неважно на какую грань и в какой точке светить (т.е. от этого зависит только ход луча)
> Вопрос вот в чем. Если мы светим (снаружи) в ту самую грань призмы, от которой отражается (изнутри) зеленый луч, пройдет ли он сквозь призму так, как я нарисовал справа?
тут мало что понял, так что отвечу как эльф: ни да, ни нет
>> No.93057 Reply
File: 445e5adf71b7234ad...
Jpg, 318.24 KB, 820×615
edit Find source with google Find source with iqdb
445e5adf71b7234adb7c622f28c59019.jpg
File: modular.jpg
Jpg, 47.14 KB, 1440×575
edit Find source with google Find source with iqdb
modular.jpg

>>93053
>>93020

Спасибо, аноны. Камень с души.
>> No.93096 Reply
>>88754
Что может считаться полем?
>> No.93100 Reply
Др/u/зья, как-то давно в винрарной советской книжке прочел о интересной гипотезе, которая говорит, что если масса Вселенной больше определенной критической, то ее нынешнее расширение когда-то замедлится, остановится, сменится сжатием, вся материя соберется в точку, будет новый Большой взрыв и все будет сначала (эта теория хорошо проиллюстрирована в 6-м сезоне Футурамы в серии Late Philip J Fry). Вот у меня вопрос: как эта теория согласуется со 2-м законом термодинамики, гласяшем,что тепловая смерть Вселенной неизбежна? Где я что упустил?
>> No.93102 Reply
>>93096
заливные луга....
>> No.93104 Reply
>>93096
Берегись батарей.
>> No.93106 Reply
>>93096
Астральное, астральное поле....светит Луна или падает снег.....
>> No.93110 Reply
>>93106
>>93102
Палишься.
>> No.93120 Reply
>>93100
Хороший вопрос. Современные теории не говорят про него ничего определённого, но например Пенроуз очень любит распинаться на эту тему. Можешь в "The Emperor's New Mind" глянуть соответствующую главу. Если вкратце, то он полагает что сингулярность на входе и сингулярность на выходе будут принципиально отличаться.
>> No.93276 Reply
File: wh_inq.jpg
Jpg, 215.55 KB, 1024×1001 - Click the image to expand
edit Find source with google Find source with iqdb
wh_inq.jpg
>>93120
я нуб в физике, поэтому
> Если вкратце, то он полагает что сингулярность на входе и сингулярность на выходе будут принципиально отличаться.
я понял как "начальные и конечные состояния системы будут принципиально различаться". Это допустимо для бытового уровня или является ересью?
>> No.93321 Reply
File: i_002.jpg
Jpg, 41.38 KB, 376×550 - Click the image to expand
edit Find source with google Find source with iqdb
i_002.jpg
>>93276
Больше атеистов в адепты Сингулярности!
>> No.93330 Reply
Правда, что фотоны можно замедлить?
>> No.93347 Reply
Фотоны нельзя. Волну - можно.
>> No.93366 Reply
>>93330
откуда инфа?
фотоны всегда движутся со скоростью света
>> No.93424 Reply
>>93366
Нет, не всегда, только в вакууме. В среде какая-нибудь хрень может двигаться быстрее.
ruwiki://Эффект_Вавилова_—_Черенкова
>> No.93434 Reply
>>93424
Фазовая скорость волны и скорость частицы не одно и тоже. Фотон летает всегда со скоростью света, просто в разных средах она разная. В вакууме его никто не сможет обскакать, если конечно Эйнштей не соврал.
>> No.93452 Reply
File: 1316383577246.jpg
Jpg, 240.74 KB, 634×982 - Click the image to expand
edit Find source with google Find source with iqdb
1316383577246.jpg
Анон, я нуб в физике, но мне надо ознакомиться с современными методами микроскопии. В частности реквестую описание применения квантовых точек в этой цели. Пытался что-то соткать из статей на вики и нихуя не вышло.
>> No.93458 Reply
File: big_FNV_VaultBoy_Dice_Painted[1].jpg
Jpg, 302.06 KB, 1084×1280 - Click the image to expand
edit Find source with google Find source with iqdb
big_FNV_VaultBoy_Dice_Painted[1].jpg
Анон, почему заряженные частицы взаимодействуют друг с другом?
>> No.93460 Reply
>> No.93461 Reply
>>93458
Это постулат.
>> No.93462 Reply
Спасибо, анон, я уже сам разобрался с квантовыми точками. Надеюсь, хоть про конфокальную микроскопию ты мне можешь кинуть хорошую годную статью для нуба?
>> No.93463 Reply
Всё, анон, я сам нашёл канал с умными людьми, просто объясняющими умные вещи.
Пойду посплю, а утром посмотрю.
http://www.youtube.com/user/ibioseminars/
>> No.93642 Reply
>>88754
Анон, посоветуй учебник/справочник по физике школьного уровня то что понадобится в университете или при сдачи ЕГЭ На школьные учебники надежды нет.
>> No.93644 Reply
>>93642
Для начала скажи свой уровень. Школьные учебники все прочитал?
>> No.93645 Reply
>>93644
Уровень девятиклассника. Собираюсь задрачивать физику самостоятельно, поэтому и спрашиваю совета.
>> No.93646 Reply
File: Безымянный222.bmp
Bmp, 76.81 KB, 160×250 - Click the image to expand
edit Find source with google Find source with iqdb
Безымянный222.bmp
>>93645
Мякишев. Все - сколько ж их там, дай бог памяти - 5 томов? Давно было дело, но учебник годный. Даже на первом-втором курсе помогал в некоторых моментах разбираться.
>> No.93673 Reply
>>93645
> то что понадобится в университете
Сперва читаешь http://yadi.sk/d/yk2jXsRA0VsHz вот эту книгу. Всю. Параллельно читаешь фейнмановские лекции по физике (все девять томов теории). И затем читаешь первые три тома Ландафшица. Скачать можно тут: http://c3v.ru/
>> No.93677 Reply
>>93673
> три тома Ландафшица
Не сложновато ли для
> девятиклассника
Эти книжки и на подготовленную голову переварить надо.
>> No.93679 Reply
>>93677
Сложно, но в физике либо знания, либо профанация. Да, для Ландау-Лифшица нужен инженерный матан, но у десятыйкласс-куна полно времени на самообразование. Думаю, книжку по математике он успеет прочитать за лето.
>> No.93684 Reply
>>93673
Огромное спасибо за 1 книгу !
Искал что то подобное. скоро выпускник, любитель физики - кун
>>93642
Почитай мой ответ по похожему вопросу, и найди порешай задачник. Но это только школьная программа...
>>93328
>> No.93711 Reply
File: 1362830229093.png
Png, 1.27 KB, 300×20 - Click the image to expand
edit Find source with google Find source with iqdb
1362830229093.png
>>93679
No.
Пусть читает пятитомник Мякишева. У него же помимо физики есть предметы, а ты ему предлагаешь самостоятельно изучить материал трёх семестров по анализу, линейной алгебре и аналитической геометрии. Это само по себе требует подготовки в виде 10-11 классов минимум.
>> No.93715 Reply
>>93711
Нет, не требует. То, что преподают на уроках математики в школах - треш и угар.
Мякишев, насколько я помню, не объясняет сути физики и даже не вводит понятия гамильтониана и лагранжиана. Почти все задачи в учебнике Мякишева - это банальное упражнение в арифметике.
>> No.93779 Reply
File: al67_ph167.jpg
Jpg, 15.48 KB, 275×300 - Click the image to expand
edit Find source with google Find source with iqdb
al67_ph167.jpg
Анон, подскажи, чем же кончилась история со сверхсветовым нейтрино? Что-то давно не слышал ничего про этот эксперимент, сказали что пересчитают, и все стихло.
>> No.93780 Reply
>>93779
Какой-то аспирант провод не до конца воткнул. Экспериментаторы почти в полном составе уволились.
>> No.93781 Reply
>> No.93782 Reply
>>93779
"Расходимся пацаны, нет превышения". Погрешность там была, связанная то ли с техникой, то ли с проводами.
>> No.93787 Reply
>>93780
>>93781
>>93782
Всем спасибо!

Жаль, я надеялся, что свет все же удасться обогнать. Ну да ладно.
>> No.94665 Reply
File: Безымянный.jpg
Jpg, 78.15 KB, 447×232 - Click the image to expand
edit Find source with google Find source with iqdb
Безымянный.jpg
Поясните пожалуйста, какие формулы они использовали и как они их связали. Я не совсем понимаю.
>> No.94666 Reply
>>94665
ruwiki://Эквивалентность_массы_и_энергии , раздел "Понятие релятивистской массы"
mrel = m/sqrt(1-v^2/c^2)
v = 0.9c
mrel = m/sqrt(1-0.81) = m/sqrt(0,19) ~= m/0.4359 ~= 2.2941m

Термин "нормальная масса" доставил.
>> No.94668 Reply
File: 941b86a48d649ef5d69705e23ef892b5.png
Png, 0.81 KB, 140×58 - Click the image to expand
edit Find source with google Find source with iqdb
941b86a48d649ef5d69705e23ef892b5.png
>>94666
Спасибо, но я так и не понял, как выводится > mrel = m/sqrt(1-v^2/c^2)
>> No.94669 Reply
>>94668
Это не выводится, это определение - релятивистской массой называется вот эта штука. Понятие релятивистской массы появилось в начале века двадцатого века. Сейчас - устарело и полностью вытеснено понятием "энергия". Вводилось для удобства тогдашних физиков, они не понимали смысла СТО. Причём введено было не Эйнштейном.
Сам Альберт первое время пользовался "инвариантной массой" и понимал под ней величину m = E0/c^2, где E0 - энергия покоя, то есть энергия, которой тело обладает в некой ИСО, относительно которой оно покоится. Потом он нашёл учебник по матану и запилил ОТО, в которой переопределил и массу, и небо, и Аллаха, но это другая история.
>> No.94858 Reply
File: Гаусс.jpg
Jpg, 235.70 KB, 576×738 - Click the image to expand
edit Find source with google Find source with iqdb
Гаусс.jpg
Привет тебе, физикач. Посоветуй хороший учебник по общей физике для подготовки к поступлению в технический вуз. Пёрышкин - унылое дно.
>> No.94859 Reply
>>94858
Читать хорошие учебники физики без знания хотя бы инженерной математики - не дело, ты их просто не поймёшь.
Сперва прочитай книгу Письменного - "Курс лекций по высшей математике", часть 1.
Тебя интересуют интегралы и особенно страницы 237-254. Самое большее за месяц ты должен прочитать эту книгу. В процессе чтения пользуйся википедией.
>> No.94862 Reply
>>94859
> хорошие учебники физики без знания хотя бы инженерной математики
В таких учебниках уже идёт отход от формул вида a=bc к более сложным, я тебя правильно понимаю?
Мне не так сильно горит - сдавать ЕГЭ буду в следующем году. Поэтому могу вдумчиво изучать его, за месяц успел бы только пробежаться.
>> No.94864 Reply
>>94862
Эта книга не стоит вдумчивого изучения. Тебе нужны только определения, умение работать с матрицами и векторным пространством и зайчатки умения применять дифференциально-интегральное исчисление в физике. Месяц, не больше.
>> No.94865 Reply
>>94864
Всё остальное можно пробежать глазами?
>> No.94869 Reply
>>94865
Нет, прочитать с применением разума.
>> No.95202 Reply
File: X0C2YvdiMxI-—-копия.jpg
Jpg, 20.46 KB, 400×207 - Click the image to expand
edit Find source with google Find source with iqdb
X0C2YvdiMxI-—-копия.jpg
Допустим, соответственно рисунку происходит удар летящего шарика известной массы и движущейся преграды вот такой вот формы условно-бесконечной массы. Удар абсолютно неупругий. Куда и как полетит шарик? Как найти вектор скорости шарика после удара, если все указанные на рисунке величины известны?
>> No.95203 Reply
>>95202
В точке удара проведи касательную. По ней он и полетит.

Разложи вектор скорости шарика до удара на векторы по касательной и по нормали к поверхности. Скорость удара по нормали погасится в ноль (абсолютно неупругий удар, после которого тела слипаются и ползут со скоростью, которую можно определить по закону сохранения импульса, то есть с нулевой, так как преграда имеет бесконечно большую массу), а скорость по касательной останется в неприкосновенности, потому что ей ничто не препятствует, шарик на самом деле не прилипает к преграде, а соскальзывает.
>> No.95206 Reply
>>95203
> скорость по касательной останется в неприкосновенности
То есть, модуль скорости шарика после удара будет равен модулю скорости до удара?
>> No.95208 Reply
>>95206
Не слушайте его, он сказал бред. Ответ получен, спасибо.
>> No.95211 Reply
>>95206
Что ж ты, товарищ, хуйню порешь? Это же основная формула при расчете брони, как же ты без нее проживешь?
>> No.95243 Reply
>>95211
А он трофейную носить будет.
>> No.95268 Reply
YouTube: Paul Scheerbart's Perpetual Wheel

Объясните, пожалуйста, анону далекому от знания физики в чем наебка.
>> No.95272 Reply
>>95268
Наёбка в том, что чувак своей энергией подпитывает установку. Видишь, он качает рычажок? Сила, которую он прикладывает, крутит колёса.
>> No.95345 Reply
>>95272
Спасибо, доброфизик.
>> No.95462 Reply
File: Imagesrr[1].jpg
Jpg, 7.09 KB, 200×236 - Click the image to expand
edit Find source with google Find source with iqdb
Imagesrr[1].jpg
Новый физика-тред. Предыдущий тред >>54970
>> No.96185 Reply
File: nuclear_physics.jpg
Jpg, 226.75 KB, 600×844 - Click the image to expand
edit Find source with google Find source with iqdb
nuclear_physics.jpg
Бамп курсом Ландау-Лифшица.
http://c3v.ru
>> No.96187 Reply
>>96185
Ух ты, альтер пиарится.

Реквестирую моар злых гениев.
>> No.96188 Reply
>>96185
Ужасный курс.
>> No.96191 Reply
File: Физика.rar
Rar, 0.87 KB, 0 files
view
Физика.rar
File: Schrodinger_by_ma...
Jpg, 167.12 KB, 600×899
edit Find source with google Find source with iqdb
Schrodinger_by_markofthedead.jpg

>>96188
Хороший курс.
>> No.96192 Reply
>>96191
Пруф.
>> No.96194 Reply
File: Fuckyeahscience_by_SamSaxton.jpg
Jpg, 141.81 KB, 899×888 - Click the image to expand
edit Find source with google Find source with iqdb
Fuckyeahscience_by_SamSaxton.jpg
>>96192
Обширнее Берклеевского курса, строже Фейнмановских лекций. Элегантно накладываются на современную математику, пример чему НМУшные лекции из архива выше.
>> No.96199 Reply
>>96194
> Обширнее Берклеевского курса
Нет. Бтв, их много, о каком идет речь?
> строже Фейнмановских лекций. Элегантно накладываются на современную математику, пример чему НМУшные лекции из архива выше.
Современный мат. аппарт квантмеха - это как раз-таки ближе к Фейнмановским лекциям, чем к Ландафшицу.

Но вообще, ещё раз посмотрел на Ландафшицу, в принципе норм. Спасибо за ссылку, буду если что пользоваться.
>> No.96201 Reply
File: strings_and_micro...
Jpg, 540.51 KB, 1000×1000
edit Find source with google Find source with iqdb
strings_and_microwaves_by_hummingvoids-d3cacch.jpg
File: quantum_physics_b...
Png, 104.27 KB, 728×585
edit Find source with google Find source with iqdb
quantum_physics_by_supertails123-d59hask.png
File: ukoku_is_like_a_b...
Jpg, 314.84 KB, 799×750
edit Find source with google Find source with iqdb
ukoku_is_like_a_boss_by_hellga-d47d3bb.jpg
File: DW___Koschei_by_s...
Png, 347.15 KB, 585×454
edit Find source with google Find source with iqdb
DW___Koschei_by_spiritedchaos.png
File: Schrodingers_cat_...
Jpg, 28.81 KB, 300×300
edit Find source with google Find source with iqdb
Schrodingers_cat__by_snowpadder.jpg

>> No.96202 Reply
>>96194
А чьи лица там? По фамилии могу назвать только Фейнмана.
>> No.96207 Reply
>>96201
Ну он кстати лучше написан. Тем не менее, берклиевский курс нацелен на широкую аудиторию, преимущественно студентов младших курсов, в то время как Ландафшиц - преимущественно 4 курс / graduate. Любой берклиевский курс такого же уровня будет современней и обширней. Особенно сейчас.
>> No.96208 Reply
>>96202
Это популяризаторы науки. Митио Каку внизу слева, Нейл Деграсс Тайсон вверху справа, Карл Саган внизу посередине, Ричард Фейнман внизу справа, Девид Аттенборо вверху посередине. Вверху слева до кучи Докинз.
>> No.96209 Reply
>>96208
Сенкс.

Эй, физики, а как насчет того чтоб устроить семинар по квантовой механике и КЭД? Выкладывайте задачки, порешаем.
>> No.96211 Reply
Анон, объясни мне дефект массы. Почему атом в "собранном" состоянии весит меньше чем в "разобранном"?
>> No.96212 Reply
>>96211
Масса эквивалентна энергии, а при распаде атома часть энергии излучается в виде, например, нейтрино.
>> No.96213 Reply
>>96212
*ядра.
fxd
>> No.96224 Reply
>>95462
друзья, кто чем будет заниматься после физфака?
>> No.96232 Reply
>>96185
перешел по ссылке. извините за оффтоп, но решил упороться книгами, что ниже в списке. Кинул Рэдволл на ридер.
>> No.96247 Reply
>>96212
Если я правильно понял, при распаде атома выделяется энергия и у конечного результата масса большеесли бы масса уменьшилась и энергия выделилась, то было бы понятно, но тут наоборот. Как это возможно?
>> No.96249 Reply
>>96247
Масса эквивалентна энергии. Частицы в ядре удерживаются за счёт энергии связей. Когда ядро распадается, частицы прекращают тратить энергию на связи, поэтому их совокупная масса покоя больше массы покоя ядра.
>> No.96261 Reply
>>96249
Теперь понятно. Просто показалось невероятным то, что масса эквивалентна энергии.
Спасибо.
>> No.96267 Reply
>>96261
Кстати об энергии.

Существует факт, или, если угодно, закон, управляющей всеми явлениями природы, всем, что было известно до сих пор. Исключений из этого закона не существует; насколько мы знаем, он абсолютно точен. Название его — сохранение энергии. Он утверждает, что существует определённая величина, называемая энергией, которая не меняется ни при каких превращениях, происходящих в природе. Само это утверждение весьма и весьма отвлечено. Это по существу математический принцип, утверждающий, что существует некоторая численная величина, которая не изменяется ни при каких обстоятельствах. Это отнюдь не описание механизма явления или чего-то конкретного, просто-напросто отмечается то странное обстоятельство, что можно подсчитать какое-то число и затем спокойно следить, как природа будет выкидывать любые свои трюки, а потом опять подсчитать это число — и оно останется прежним.

Фейнман.
>> No.96275 Reply
File: Магнит.JPG
Jpg, 22.78 KB, 494×280 - Click the image to expand
edit Find source with google Find source with iqdb
Магнит.JPG
>> No.96636 Reply
>>88754
Физчан, поясни по квантмеху(предполагаю, что по квантмеху) следующую ситуацию. Есть (я вольно цитирую нашего препода) тонкостенная плоскопараллельная пластинка на которую падает набор параллельных одной частоты лучей света. Каждый из лучей отражается и преломляется одновременно, в результате происходит дифракция Фраунгофера. Вопрос в следующем: как, блядь, луч может преломляться и отражаться одновременно на границе со средой? Если это всё-таки возможно из-за неровностей поверхности, как объяснить то, что энергия каждого луча после взаимодействия со средой возрастает, т.к. складывается из энергии отраженного луча, которая равна энергии исходного, и энергии преломленного?
>> No.96647 Reply
>>96636
> энергия каждого луча после взаимодействия со средой возрастает
инфа-то хоть 100%?
>> No.96675 Reply
Товарищи-физики, а проективное гильбертово пространство квантовых состояний и пространство Фока это один и тот же зверь или есть тонкая разница?
>> No.96718 Reply
>>96647
Не-а. Это лишь мои предположения. Хочу пояснений мудрых, но, видимо, уйду нахуй.
>> No.96750 Reply
>>96636
> как, блядь, луч может преломляться и отражаться одновременно на границе со средой?
нет, это не из-за неровностей и даже не квантмех. Это волновая оптика, детка. Кури принцип Гюйгенса. В случае плоской волны на границе раздела сред есть джва варианта распространяющегося далее волнового фронта - отраженный и преломленный. Кстати, тот-самый-закон-преломления следует оттуда же. Всегда реализуются оба варианта, кроме случая полного внутреннего отражения. в этом случае просто не получается волнового фронта, ибо вторичные волны движутся быстрее точки контакта изначальной волны с поверхностью.
> как объяснить то, что энергия каждого луча после взаимодействия со средой возрастает
не каждого, а только некоторых, в этом и есть суть дифракции, йуный падаван. Складывается энергия отраженного одного луча и преломленного другого луча, так что всё правильно. А в некоторых точках, наоборот, происходит деструктивная интерференция и энергия обращается в нуль. Такие дела.
>> No.96774 Reply
>>96750
Я несколько неправильно выразился, прошу прощения. Когда спрашивал, почему
> энергия каждого луча после взаимодействия со средой возрастает
я подразумевал, что этот лучик до этого на два поделился(отраженный и преломленный), а поскольку считал, что отраженный луч ничем, в том числе и энергией, от падающего не отличается, думал, что его энергия, как сумма энергий преломленного и отраженного лучей, ему соответствующих, возрастает. В итоге я всё равно нихуя ничего совсем полностью абсолютно совсем не понял из твоего объяснения, ну да и ладно, хоть про принцип Гюйгенса почитаю. Спасибо за неравнодушие ня.
>> No.96788 Reply
>>96675
За уже несколько лет изучения квантовой механики/КТП не было необходимости строго различать эти термины. Но я себе это представляю так - пространство состояний любой квантовой системы является (постулат) гильбертовым (ну и проективным, поскольку вектора, отличающиеся только длиной, соответствуют одному состоянию). А пространство Фока - это пространство состояний какой-то системы (обязательно гильбертово, по постулату) со специальным базисом. Каждый из базисный векторов пространстве Фока соответствует состоянию, в котором есть определённое число частиц каждого типа. Т.е. например стандартный базис, используемый при описании осциллятора |n> и есть фоковский базис для гильбертова пространства состояний осциллятора: в состоянии |n> находится ровно n квантов~n частиц. Бывают ли системы, в пространстве состояний которых нельзя выбрать фоковский базис я не уверен, но думаю что бывают. Например моё (правда сильно научно-популярное) знание конформной теории поля говорит, что в ней нет никаких "частиц", а потому наверное нет и базиса, состоящего из состояний с определённым числом частиц. Но тут я мог быть во всём неправ.
>> No.99303 Reply
Привет, анон. Хочу обмазаться базовой астрономией, чтобы потом углубляться. Нужна литература. Что посоветуешь? Чую, что придется параллельно обмазываться физикой, курить теорию относительности, магнетизм и т п. Англоязычную литературу предлагать можно и желательно.
>> No.99305 Reply
>>99303
На рутрекере есть курс 'Introduction to astronomy' от Coursera.org.

Какой бэкграунд в этой самой физике? На уровне техвуза?
>> No.99308 Reply
>>99305
Спасибо!
Я вообще гуманитарий, все еще хранящий в сердце любовь к точности. Так что бэкграунд - безупречно сданный экзамен в 9 классе. можно смеяться
Так что мне придется начать изучать все с азов.
>> No.99309 Reply
>>99305
Книги пожалуйста.
Кстати наблюдать за звёздами через комп возможно ли?
Есть ли программы для этих дел? (для *nix)
В моей местности над городом облако смога, звёзды плохо видно, да и телескопа нету у меня.
>> No.99346 Reply
>> No.100751 Reply
Анон, а как электрический ток "понимает", что цепь замкнута. Допустим, что есть источник питания и цепь, соединяющая плюс и минус. Разомкнём цепь - тока нет. Сомкнём - он пошёл.

Я себе представляю это по-детски как-то: ток пошёл от плюса, дошёл до места разрыва, следовательно дальше пойти не может, возвращается назад и в минус не попадает. Но это не так разумеется, а может и так. в школьные годы совсем этот вопрос не осветили, все приняли за данность, что вот ток идёт, когда цепь замкнута, и не идёт, когда разомкнута Ещё раз, как ток "понимает", что можно дойти до минуса?

Почему в проводниках всегда есть свободные электроны? Насколько я знаю, ток при движении по медному проводу, напимер, захватывает за собой свободные электроны, находящиеся в пустотах между... атомами. Откуда им потом обратно взяться? Не отделяться же от атомов. Тоже как-то смутно представляю.

И что случается со всем этим потоком электронов, когда он доходит до цели? Включил я например настольную лампу. Ток накаляет спиральку а после того, как "наколол" куда этот ток уходит. Я вытащил вилку лампы из розетки и куда девается весь тот ток, оставшийся в проводе лампы?

Век электричества, а я нихуя не понимаю. Объясни мне анончик что-нибудь пожалуйста.
>> No.100753 Reply
>>100751
Электрический ток это движение заряженных частиц, Например электронов или ионов. А движутся они исходя из направления своего заряда - в случае с электронами они движутся от точки с отрицательным потенциалом к точке с положительным потенциалом, т.к. имеют заряд с отрицательным знаком. Когда в проводнике отсутствует разность потенциалов заряженные частицы никуда направленно не движутся, соответственно тока нет. Большинство металлов имеют электронную проводимость, в связи с тем, что электроны довольно легко перемещаются между атомами. Алсо чем сложнее оторвать электрон от атома тем выше сопротивление материала, в случае электронной проводимости.
Кстати электроны не обязательно должны двигаться по твердым средам. Отличный пример - ЭЛТ или вакуумные лампы. Там при помощи подогревателя вокруг катода создается облако электронов, которое позволяет при приложении достаточной разности потенциалов между анодом и катодом образовать токопроводящий канал.
Как то так. Не претендую на истину, но вроде грубых ошибок я не допустил.
>> No.100755 Reply
>>100753
> при помощи подогревателя вокруг катода создается облако электронов
Точнее говоря нагретый катод испускает электроны.
мелкофикс
>> No.100756 Reply
>>100751
Если коротко, насколько я понимаю, электроны притягиваются к положительному потенциалу. А если цепь не замкнута, то их ничего не притягивает, и они стоят (соответственно, если цепь размыкают, они остаются на тех же местах, на которых остановились).

мимошкольник
>> No.100758 Reply
>>100756
>>100753
Что собой представляет положительный или отрицательный потенциал? Тоже смутное представление, что даже высказывать его стыдно.
>> No.100759 Reply
>>100758
По-моему это что-либо, что заставляет электроны двигаться. А заставляет тем, что имеет отрицательный или положительный заряд. Например, если двигать магнит рядом с замкнутым контуром, то там возникнет индукционный ток. А если контур разомкнут, на его концах возникнет разность потенциалов. То есть отрицательный заряд толкает электроны на другую сторону, к положительному, а они не могут туда попасть, потому что там разрыв, поэтому они будут ждать, пока цепь не замкнется.
Не знаю, насколько правильно и понятно это сформулировано, но надеюсь, что основной смысл верен.

тот же мимошкольник
>> No.100764 Reply
>>100756
> А если цепь не замкнута, то их ничего не притягивает, и они стоят (соответственно, если цепь размыкают, они остаются на тех же местах, на которых остановились)
Нет! Если цепь незамкнута, то их продолжает тянуть, но они не могут пройти через разомкнутое пространство.
Смотри, например, случаи пробоя через диэлектрик или газ, когда накладывается достаточно сильный потенциал (= "достаточно сильное напряжение в цепи").
>> No.100766 Reply
>>100764
Да, я понял это, когда пытался ответить на этот >>100758 вопрос.
>> No.100767 Reply
>>100751
> Анон, а как электрический ток "понимает", что цепь замкнута. Допустим, что есть источник питания и цепь, соединяющая плюс и минус. Разомкнём цепь - тока нет. Сомкнём - он пошёл.
Точно так же, как вода в дальнем конце заполненной трубы понимает, что в другом её конце открыли кран и можно теперь двинуться к месту вытекания.

Давление воды (в отличие от самих её материальных частиц) передаётся практически мгновенно. И точно так же передаётся по электрической цепи потенциал, когда мы её размыкаем/смыкаем.
>> No.100770 Reply
>>100751
Когда между концами проводника возникает разность потенциалов (напряжение), электрическое поле распространяется по проводнику с околосветовой скоростью. Поле действует на заряд. Так электроны и "узнают", что нужно двигаться. Так как электроны имеют отрицательный заряд, то они движутся против линий поля.
>> No.101074 Reply
Физикоанон, почему физические величины выражают именно вещественными числами?
>> No.101321 Reply
Прошу знающих пояснить за кинематику сложного движения. Долгое время два семестра не могу уложить в голове очевидные, наверное, вещи.

Есть две системы координат, абсолютная и относительная. Если мы их вводим искусственно, то чтобы найти абсолютное ускорение, надо сложить вектора относительного, переносного и кориолисова ускорения.

За переносное ускорение берется ускорение в абсолютной системе отсчета точки, соответствующей исследуемой точке. Если так(тут я всё время путаюсь и иногда, кажется, понимаю), то чем это ускорение отличается об результирующего абсолютного?
>> No.101337 Reply
>>101321
Бегаю по движущемуся кораблю, то очевидно что мое ускорение не будет равно ускорению корабля. Или не очевидно?
>> No.101345 Reply
>>101337
Не, на словах-то всё очевидно. Заковыка в применении именно формул для сложного движения точки.
Даже могу точнее сформулировать: когда находим переносное ускорение, то что это такое? Ускорение начала координат для подвижной системы отсчета в неподвижной. А т.к. подвижная система координат совпадает с точкой, у меня происходит бамплимит.
  
Wa=We+Wr+Wc
>> No.101379 Reply
>>100758

Потенциал - это такая удобная величина. Каждая точка пространства имеет потенциал и каждое значение потенциала характеризует некоторое множество точек. при перемещении заряда из любой такой точки в другую суммарная потраченная энергия будет равна нулю. при этом разность потенциалов любой пары точек соответствует энергии, необходимой для переноса единичного заряда из одной в другую. Т.е. численное значение потенциала особого смысла не имеет, хотя для удобства потенциал "на бесконечности" часто приравнивают к нулю и относительно него считают все остальные.
>> No.101380 Reply
>>101379
А что такое энергия?
>> No.101394 Reply
>>101379
альсо, потенциал бывает не только электрический. Если есть такое поле, что сила, с которой оно воздействует на произвольное тело, зависит только от его координат и пропорциональна некоторой величине k, которое является характеристикой этого тела, то можно аналогичным образом определить потенциал данного поля, но вместо заряда будет k.
>> No.101407 Reply
>>101345
Вообще говоря, если точка совпадает в каждый момент с некоторой координатой подвижной системы, то нам нет необходимости что-либо городить (абсолютное ускорение совпадает с переносным, а относительное и кориолисово ускорения нулевые). Проблемы начинаются, когда точка со временем меняет свою координату в подвижной системе. Так вот, фишка в том, чтобы зная как двигается подвижная СК относительно неподвижной и как двигается точка в подвижной СК найти как двигается точка в неподвижной СК.
>> No.101439 Reply
>>101074
Экспериментальный факт. Но вообще это как посмотреть, некоторые величины очень удобно вводить как комплексные числа. Период колебаний, например.
>> No.101441 Reply
Кто-нибудь сталкивался с пространствами нецелых размерностей? Я понимаю что фракталы, бла-бла, Хаусдорфова размерность. Было бы интересно построить в них какую-нибудь физику, хотя бы законы Ньютона, и посмотреть как они работают.
>> No.101442 Reply
>>101439
Какой экспериментальный факт, няша?
>> No.101443 Reply
>>101442
Такой, что все наблюдаемые величины оказываются вещественными.
>> No.101444 Reply
>>101443
На каком основании делается вывод, что они вещественные?
>> No.101445 Reply
>>101444
"Вещественные" и "комплексные" - не более чем математическая абстракция. Если наблюдаемая величина ведёт себя как вещественная (то бишь непрерывно меняется и линейно упорядочена), то мы и рассматриваем её как вещественную. А величин, которые ведут себя как комплексные (с нужными нам законами сложения/умножения) вокруг не видно.
>> No.101446 Reply
>>101445
> то бишь непрерывно меняется и линейно упорядочена
Любое бесконечное множество можно упорядочить. А почему мы требуем непрерывность, а не плотность?
>> No.101447 Reply
>>101446
> Любое бесконечное множество можно упорядочить.
Угу, только у нас порядок сохраняется при сложении и умножении на скаляр.
> А почему мы требуем непрерывность, а не плотность?
Потому что с плотными, но не непрерывными множествами нужно очень долго трахаться, чтобы построить анализ. Оно нам надо?
>> No.101448 Reply
>>101447
> только у нас порядок сохраняется при сложении и умножении на скаляр
Комплексные числа и рациональные числа тоже образуют поля.
> Потому что с плотными, но не непрерывными множествами нужно очень долго трахаться, чтобы построить анализ. Оно нам надо?
А разве мы отождествляем явления с математическими объектами произвольно?
>> No.101449 Reply
>>101448
> А разве мы отождествляем явления с математическими объектами произвольно?
Ви таки будете смеяться, но да. Работает - отождествили. Один хрен все наши теории на данный момент эффективные, а не фундаментальные.
> Комплексные числа и рациональные числа тоже образуют поля.
Порядок на комплексных числах не уважает сложение, увы.
>> No.101450 Reply
>>101449
> Работает - отождествили.
А как мы тогда можем узнать, что оно "работает"?
> Порядок на комплексных числах не уважает сложение
На комплексных числах нет общепринятого порядка же. По-моему, можно ввести такой, который уважает.
>> No.101451 Reply
>>101450
> А как мы тогда можем узнать, что оно "работает"?
Результаты правильные получаются. Самолёты летают, данные по проводам передаются, капча двачуется.
> По-моему, можно ввести такой, который уважает
Я был уверен, что нельзя. Если найдёшь - поделись, мне интересно.
>> No.101452 Reply
>>101451
> Результаты правильные получаются.
Результаты получаются, с этим я не спорю, но что значит "правильные"?
> Я был уверен, что нельзя. Если найдёшь - поделись, мне интересно.
Хм. Как насчёт лексографического?
z1 = a + ib;
z1 < z2 iff a<c OR (a=с AND b<d);
>> No.101453 Reply
>>101452
z1 = a + ib
z2 = c + id
z1 < z2 iff a<c OR (a=с AND b<d)
То есть.
>> No.101456 Reply
>>101452
> что значит "правильные"
У нас же есть с чем сравнивать: реальность. Эксперимент.
> Как насчёт лексографического?
Сравнение по проекции на вещественную ось по сути? Ну да, ты прав, сложение и умножение на скаляр сохранят порядок. Видимо, основное свойство комплексных чисел - умножение друг на друга, иначе можно рассматривать две вещественные наблюдаемых вместо одной комплексной. А вот ещё и умножение с порядком подружить не получится.
>> No.101457 Reply
>>101456
> У нас же есть с чем сравнивать: реальность. Эксперимент.
Но как истолковывать полученные математические сущности? Как понимать результаты экспериментов?
>> No.101458 Reply
>>101457
Математические сущности дают тебе цифру. Стрелка на приборе показывает тебе цифру. Ты сравниваешь две цифры и понимаешь, правильна твоя теория или нет.
>> No.101459 Reply
>>101458
> Математические сущности дают тебе цифру.
Число. Нет, не всегда.
> Стрелка на приборе показывает тебе цифру.
А как я наношу шкалу на прибор, м?
>> No.101460 Reply
>>101459
> Нет, не всегда
Это входит в твою свободу построения теории. Хочешь - она будет давать тебе число, хочешь - ещё что-нибудь. Просто с числами работать наиболее удобно.
> А как я наношу шкалу на прибор
Собственно, как хочешь, так и наносишь. Главное чтобы результат совпал с теорией. Не стоит пугаться, что мы подгоняем шкалу под теорию: любой прибор меряет не один эксперимент а множество, и нас интересует лишь их взаимоотношение. Градуировка шкалы весов по известным массам, допустим, тоже эксперимент по сути.
>> No.101461 Reply
>>101460
Гм. То есть, скажем, комплекснозначное время никто не рассматривает просто потому, что никто не видит в этом смысла?
>> No.101462 Reply
>>101461
Ну да. Время, судя по нашим наблюдениям, не обладает свойствами, ради описания которых стоит рассматривать его как комплекснозначную величину.
Я тебе сейчас рассказываю позитивистский взгляд на физику, который ещё известен как "заткнись и считай". Он мне нравится простотой и конкретностью, но вполне возможно что при таком философском подходе что-то упускается. Можешь популярные книги Девида Дойча почитать для рассмотрения с другой стороны
>> No.101464 Reply
>>101462
А разве есть наблюдения, которые заставляют считать, что время обладает свойствами, для выражения которых нужны вещественные числа?
> книги Девида Дойча
Спасибо, иду качать. Буду рад, если ещё что-нибудь посоветуешь. inb4 "О науке" Пуанкаре - читал
>> No.101465 Reply
>>101464
> А разве есть наблюдения, которые заставляют считать, что время обладает свойствами, для выражения которых нужны вещественные числа?
Непрерывно (на первый взгляд, да), обладает порядком и мы можем сравнивать его промежутки между собой. Забыл, как в математике последнее свойство формулируется.
> Буду рад, если ещё что-нибудь посоветуешь
Про философию науки? Каких-нибудь очевидных Поппера с Гейзенбергом. Можешь ещё Пенроуза почитать, не совсем о том но занятно.
>> No.101467 Reply
>>101465
> Непрерывно (на первый взгляд, да)
Мы мысленно можем разделить любой кусочек времени надвое - это свойство плотности, оно и у рациональных чисел есть. А непрерывность - это когда в любом ограниченном подмножестве содержится точная грань, и вот наличие этого свойства у времени совсем не очевидно.
> Каких-нибудь очевидных Поппера с Гейзенбергом.
А есть что-нибудь поновее? Что-нибудь из области феноменологии?
>> No.101468 Reply
>>101467
> непрерывность - это когда в любом ограниченном подмножестве содержится точная грань
Есть математика, влияющая на физику (читай - результаты), а есть не влияющая. Вот разница между плотностью и непрерывностью из последних. Наверняка можно, если очень захочется, построить теорию в которой это будет играть роль. Но сейчас таких даже близко не рассматривают, насколько я знаю. Поэтому все берут непрерывность, так как с ней проще.
> А есть что-нибудь поновее? Что-нибудь из области феноменологии?
Не знаю, увы.
>> No.101469 Reply
>>101468
Оки, спасибо.
>> No.101779 Reply
File: IMG_20130527_214848.jpg
Jpg, 81.56 KB, 800×600 - Click the image to expand
edit Find source with google Find source with iqdb
IMG_20130527_214848.jpg
Привет товарищам-физикам с кафедры математики. У нас поднялся один вопрос, решили, что лучше будет спросить здесь:
Допустим, у нас есть шестигранная игральная кость. Представим, что она представляет собой идеальный куб, пренебрегая как закруглёнными углами, так и точками-углублениями.
Мысленно разделим нашу кость на 27 кубиков равного объёма. Допустим, нам известна средняя плотность каждого кубика ρ1, ρ2, ..., ρ27. Можем ли мы, основываясь на этой информации, высказать какие-нибудь предположения о распределении вероятности выпадания конкретной стороны этой кости при её метании?
>> No.101784 Reply
>>101779
Сперва нужно найти центр масс. Центр масс дает точку приложения силы тяжести. Еще наверное неплохо бы найти тензор инерции. Затем нужно построить конфигурационное пространство кубика. По каждой точке этого пространства (набор некоторых начальных условий) можно рассчитать грань кубика, которая выпадет в результате. Помимо чистой динамики (которая будет выражаться 6 степенями свободы) нужно будет как-то учитывать диссипацию энергии. Потом нужно проинтегрировать все это дело (распределение граней по точкам пространства), с учетом знания примерной вероятности выпадения соответствующих начальных условий.
Вот такое мое представление об этой задаче.
>> No.101785 Reply
>>101784
Оу-воу, тут происходит что-то интересное. Можно подробнее, няша? Как найти центр масс, как искать тензор инерции и как строить конфигурационное пространство? И что такое диссипация энергии?
другой анон
>> No.101788 Reply
>>101784
Хотя конечно это такой численный эксперимент получается. Можно попробовать проще, на интуитивном представлении - чем ближе центр масс к соответствующей грани, тем больше вероятность ее выпадения. Можно попробовать считать устойчивость равновесия кубика, лежащего на соответствующей грани. Чем выше устойчивость, тем больше вероятность выпадения этой грани. Можно считать и неусточивость различных положений кубика, когда он соприкасается со столом в точке или по прямой. Чем больше неустойчивость в сторону определенной грани - тем выше вероятность ее выпадения.
>> No.101803 Reply
>>101779 Прозреваю, что легко можно напороться на что-то вроде парадокса Бертрана, т.к. у нас очень расплывчато сформулировано, как и на что мы бросаем кубик. На самом деле, похожая задача предлагалась на одном студенческом конкурсе, и там народ толком ни к чему и не пришёл.
>> No.102916 Reply
Анон, я тут пытаюсь осилить книжку про введение в небесную механику некоего Fitzpatrick, и что-то я на первой же задаче срезался. Можете помочь?
Consider an isolated system of N point objects interacting via gravity. Let the mass and position vector of the ith object be mi and ri, respectively. What is the vector equation of motion of the ith object? Write expressions for the total kinetic energy, K, and potential energy, U, of the system. Demonstrate from the equations of motion that K+U is a conserved quantity.
>> No.102926 Reply
>>102916 С чем у тебя проблемы: с переводом или с каким-то из пунктов решения?
>> No.102933 Reply
>>102926
Со всем решением. Даже не знаю, как подступиться, честно, хотя в калькулус вроде могу. Я записываю силы, действующие на i-тое тело и... все. Я не могу взять отсюда интеграл и записать уравнение движения в явном виде, и даже уравнение скорости. И откуда здесь брать потенциальную энергию? Как потенциальную энергию взаимодействия?
>> No.103073 Reply
Суп, анончик. Через неделю у меня экзамен по электродинамике и магнетизму, а я почти ничего не знаю. Как быть, что почитать? Реквестирую лучшие учебники, чтобы все сразу стало понятно, и прочую литературу.
>> No.103139 Reply
>>102933
Джентльмены, уже целый день прошел, а у меня по-прежнему нет идей. От вас помощи не ждать?
>> No.103146 Reply
>>103139
Ну допустим напишем шестимерный интеграл функции, которая в зависимости от позиции кубика будет однозначно давать ответ, на какую сторону упадет. Выглядит довольно очевидно, но в реализации очень сложно.
>> No.103154 Reply
>>102916
Это разве не расширенная проблема трех тел? Она же нерешаема.
>> No.103156 Reply
>>103154
Проблема трёх тел решается введением комплексного времени.
мимо
>> No.103159 Reply
>>103156
Упорот? Гравитационная задача N тел не решаема.
>> No.103160 Reply
>>103159
Если время - комплексное число, то решаема.
обратно мимо
>> No.103161 Reply
>>103156
Соус можно? Я слышал про решение задачи трёх тел в истинно математическом смысле: кем-то был получен очень хреново сходящийся ряд, непригодный для расчётов.
>>102933
> И откуда здесь брать потенциальную энергию? Как потенциальную энергию взаимодействия?
Именно, больше неоткуда же.
>> No.103164 Reply
File: софья-ковалевская.pdf
Pdf, 0.30 KB, 595×842 - Click the image to get file
софья-ковалевская.pdf
>>103161
Кто-то допилил идеи Ковалевской, соус лень гуглить.
>> No.103165 Reply
>>103161
А с N тел и гравитацией что делать?
>> No.103166 Reply
>>103165
Потенциальная энергия же складывается. Пишешь для двух тел, суммируешь по всем парам.
>> No.103167 Reply
>>103166
Да нет, я про уравение движения, vector equation of motion.
>> No.103194 Reply
>>103167
Ровно аналогично. Силы от разных тел складываются, останется только второй закон ньютона записать.
>> No.103199 Reply
>>103194
Это было первым, что я сделал.
xi''=GMisum(mj/(xi-xj)^2),1<=j<=N,j!=i.
И как мне отсюда уравнение движения посчитать? Боюсь, я не осилю решать это чудо, особенно учитывая то, что остальные тела под действиям притяжения тоже движутся и xj меняется в зависимости от xi.
>> No.103228 Reply
>>103199
Это и есть уравнения движения, бака. Никто не просит тебя их решать.
>> No.103247 Reply
>>103199
Consider an isolated system of N point objects interacting via gravity. Let the mass and position vector of the ith object be mi and ri, respectively. What is the vector equation of motion of the ith object? Write expressions for the total kinetic energy, K, and potential energy, U, of the system. Demonstrate from the equations of motion that K+U is a conserved quantity.
Ну по порядку:
> What is the vector equation of motion of the ith object?
F = ma, где
cумма гравитационных сил
> Write expressions for the total kinetic energy, K, and potential energy, U, of the system.
K = сумма кинетических +1/2(сумма потенциальной энергии для каждого тела в поле всех остальных тел, то есть для i будет с m[i]m[j]/r - сумма по j)
> Demonstrate from the equations of motion that K+U is a conserved quantity.
Ну F = -grad U и, грубо говоря,
-(grad U) * v = m v'*v
-dU = d(mv^2/2)
d(U+mv^2/2)=0 индексы и значочки суммы сам расставишь.
>> No.103252 Reply
>>103073
бамп
>> No.103295 Reply
>>103228
А. Окей. Тогда ладно, тогда все просто. Я-то думал, имеется ввиду именно уравнение движения в явном виде найти, а надо просто второй закон Ньютона написать.
Спасибо большое.
>> No.103317 Reply
>>103295
Это и есть явный вид уравнений, если подставить вместо F -сумму сил действующих на тело, ну и в правой части записать m[i]*v'[i]. Сколько их всего, алсо?
>> No.103625 Reply
File: Hina.PNG
Png, 1929.96 KB, 1155×697 - Click the image to expand
edit Find source with google Find source with iqdb
Hina.PNG
Бамп с почётной второй страницы.
Я хочу овладевать теорией струн и другими космологическими моделями.
Первый мой вопрос - какие разделы математики и физики нужно повторить?
И ещё - можете накидать литературы по которой лучше всего это изучать?
Заранее спасибо.
>> No.103636 Reply
>>103625
Пока не будешь понимать целиком "Зеркальную симметрию и алгебраическую геометрию" Кокса и Катца, не приходи.
А вообщ есть такой сайт http://www.staff.science.uu.nl/~hooft101/theorist.html
И еще вот такой сайт http://stringworld.ru/
>> No.103637 Reply
>>103625
Двухтомник Виттена. Там же будет и введение в диф.гем, описание тетрадного формализма в гравитации Еще посмотри курсы стекловки, может, там было что-то по теме, мне кажется.
>> No.103640 Reply
>>103636
>>103637
О, большое спасибо. Наверное с первого тома и начну, он вроде не показался мне сложным.
>> No.103676 Reply
>>103625
http://abstrusegoose.com/272
удваиваю стрингворлд
>> No.103702 Reply
>>103625
Я бы порекомендовал Смолина "Неприятности с физикой", если еще не читал. Занимательная вещь.
>> No.103727 Reply
>>103702
я бы не строго не рекомендовал Смолина. Посмотри его последние работы на архиве - очевидно, что это шарлатан. Такая-то халтура. Пишет увлекательно, но в теоретической физике ни черта не понимает.
>> No.103751 Reply
>>103727
> Посмотри его последние работы на архиве - очевидно, что это шарлатан
А по-моему норм. Читал его статьи про квантовую гравитацию, математику подробно не осилил но идея-то хорошая. Можно примеры очевидного шарлатанства?
>> No.103767 Reply
Физики, объясните мне, пожалуйста, в чем смысл теоремы Нетер.
>> No.103785 Reply
>>103767
Смысл в том, что любой симметрии отвечает закон сохранения какой-то величины. каков вопрос-таков ответ
>> No.103790 Reply
>>103727
Да разве ж так бывает, чтоб увлекательно и понимает? Если книгу описывают как экстра понятную, увлекательную и интересную, то я сразу начинаю думать, что наверняка это бесполезный мусор. Необходимо искать и читать как можно более глубокие, точные и актуальные талмуды, остальные характеристики книги как понятность языка и т.п. - это вообще роскошь непозвольительная для спеца.
мимокрок
>> No.104283 Reply
File: 1293429483_gifki_03.gif
Gif, 407.22 KB, 150×240 - Click the image to expand
edit Find source with google Find source with iqdb
1293429483_gifki_03.gif
Физики, объясните, пожалуйста, из-за чего частота колебания груди в два раза больше частоты прыжков? Есть ли этому физическое объяснение в рамках кинематики? Я в растерянности.
>> No.104304 Reply
>>104283
Физика не отвечает на вопрос "почему", физика отвечает на вопрос "каким образом". Дай мне тян для опытов - выведу уравнение. По гифке ничего не скажу.
>> No.104306 Reply
>>104283
Амплитуда и инерция меньше, вдобавок сказывается собственная упругость груди помимо собственно силы тяжести.
>> No.104314 Reply
>>104304
Ты ненастоящий физик.
>> No.104881 Reply
http://imperium.lenin.ru/~verbit/MATH/programma.html
Есть что-нибудь аналогичное для физики?
>> No.104882 Reply
File: 94317473_1_644x461_landau-ld-lifshits-em-teoretich.jpg
Jpg, 23.52 KB, 644×393 - Click the image to expand
edit Find source with google Find source with iqdb
94317473_1_644x461_landau-ld-lifshits-em-teoretich.jpg
>> No.104883 Reply
>>104882
Слишком легко.
>> No.104886 Reply
>> No.104892 Reply
>>104886
> http://www.staff.science.uu.nl/~hooft101/theorist.html
Пфффф. Ландафшиц плюс струнная ересь теория.
>> No.104894 Reply
>>104886
Неужели в физике так мало всего? Прочитал Ландавшица - и работай на нобелевку?
>> No.104896 Reply
>>104894
А то. Удачного построения квантовой гравитации.
>> No.104899 Reply
>>104896
Нет, правда? Так просто? Я не верю. Ландау умер полвека назад. Должно же с тех пор в физике хоть что-то произойти?
>> No.104900 Reply
>>104881
http://www.staff.science.uu.nl/~hooft101/theorist.html
Инструкция от нобелевского лауреата. Заголовок сразу намекает.
>> No.104901 Reply
>>104899
Понимаешь ли, Ландау был последним учёным, который знал всю теоретическую физику. С тех пор она распалась на кучу отдельных частей, среди которых выделяются упоротые струнные теоретики и ещё более упоротые матфизики, богатые твердотельщики, куча феноменологов всех сортов и расцветок, занимающиеся термоядерными реакторами специалисты по плазме и ещё хрен пойми кто. Что происходит во всём этом зоопарке я без понятия.
>> No.104903 Reply
>>104901
Вот этот зоопарк и представляет наибольший интерес же. Кто есть кто? Кто чем занимается? Кто чего достиг за последние пять лет?
>> No.104904 Reply
>>104903
Так никто не знает же. В лучшем случае тебе расскажут про события в области, которой занимается рассказывающий. Что в каком-то разложении чего-нибудь наконец посчитали пятый член.
>> No.104905 Reply
>>104904
> В лучшем случае тебе расскажут про события в области, которой занимается рассказывающий.
Это весьма годный вариант. Чтобы составить полную картину, достаточно составить список наличествующих разделов физики и выбрать по одному представителю из каждого раздела.
>> No.104906 Reply
>>104905
В современной науке каждый человек занимается весьма узкими проблемами, Ландау и Фейнманы перевелись. Тебе потребуется много человек.
>> No.104911 Reply
>>104906
Не беда. Думаю, надо начать с составления списка живых и интересных разделов физики. Поможешь?
>> No.104927 Reply
>>104911
Давай, только не сейчас
>> No.104928 Reply
>>104927
Хорошо, я подожду. Спасибо.
>> No.104941 Reply
Интересная новость: >>b/2337115
>> No.105014 Reply
Подскажите, пожалуйста, годную книгу по общей физике. В школе протупил, теперь буду повторять. Желательно с задачами и примерами их решений.
>> No.105015 Reply
>>105014
Савельев, курс общей физики. Внимательно изучи вводные математические параграфы, без них ты ни слова не поймёшь дальше.
>> No.105022 Reply
>>105015
Спасибо.
>> No.105139 Reply
Означает ли это, что информацию можно передавать быстрее скорости света? Отсутствие изменений - 0, изменение - 1.
http://lenta.ru/news/2012/11/07/nanowaveguide/
>> No.105140 Reply
>>105139
Нет, конечно. Тебе же в конце написали "Авторы подчеркивают, что при этом не происходит нарушения фундаментального запрета на перенос информации и энергии быстрее скорости света. Речь в данном случае идет не о скорости энергии света, а о движении его фазы, которое может быть любым (аналогично скорости движения солнечного зайчика)."
Понимаешь, что имеется в виду под скоростью движения солнечного зайчика? Она может быть выше скорости света, но информация при этом не передается со скоростью выше света.
>> No.105141 Reply
>>105140
> Понимаешь, что имеется в виду под скоростью движения солнечного зайчика?
Нет. Что?
>> No.105142 Reply
>>105141
Представь себе лазер, находящийся на Луне. Направим на Землю и включим. Из-за ограничений скорости света он достигнет Земли только через некоторое время, однако непосредственно точку контакта с Землей можно перемещать быстрей скорости света, достаточно быстро наклонить лазер. Ну с солнечным зайчиком тоже самое, только там не лазер, а отраженный от зеркала луч.
>> No.105144 Reply
>>105142
> непосредственно точку контакта с Землей можно перемещать быстрей скорости света
Изменение наклона лазера вызовет мгновенное изменение положения точки контакта с Землёй?
>> No.105145 Reply
>>105144
Нет, изменение передастся со скоростью света. Выше скорости света будет именно движение точки. Можешь почитать на википедии "Сверхсветовое движение", там есть подобные примеры.
>> No.105146 Reply
>>105145
> Выше скорости света будет именно движение точки.
Но почему? Движение относительно чего?
>> No.105147 Reply
>>105146
Давай увеличим масштабы. Пусть будет расстояние от лазера до бесконечно большой плоскости 1 световая минута. Включи лазер и опиши дугу 90 градусов за одну секунду. Тогда только через минуту на плоскости появится точка, которая будет двигаться с колоссальной скоростью, выше скорости света.
>> No.105148 Reply
>>105147
> точка, которая будет двигаться с колоссальной скоростью
Относительно чего?
>> No.105149 Reply
>>105148
И относительно любой точки плоскости, и относительно самого лазера.
>> No.105150 Reply
>>105148
За одну секунду на плоскости точка пройдет расстояние в +- световую минуту. Но несмотря на это, от подачи луча все равно пройдет ровно минута.
>> No.105151 Reply
>>105149
Чтобы точка изменила положение, нужно, чтобы фотоны дошли от лазера до плоскости.
Допустим, есть лазер, который светит в точку плоскости 1. Мы поворачиваем лазер на какой-то угол, теперь выходящие из него фотоны направлены в точку 2. Точки 2 они могут достичь не быстрее чем за c.

Разве движение "зайчика" на плоскости непрерывно? Разве зайчик пробегает все точки между 1 и 2?
>> No.105153 Reply
>>105151
Да, непрерывно. В процессе движения лазер же испускает фотоны. На счет всех точек между 1 и 2 сложно сказать однозначно, просто потому как фотонов не бесконечное число. Но по крайней мере у него будет траектория между 1 и 2.
Вот смотри, ты сперва светишь 1 секунду в 1 точку, потому равномерно поворачиваешь лазер в течение секунды до точки 2. Тогда через с на плоскости зайчик будет в 1, после этого в течение секунды переместится до 2.
>> No.105154 Reply
>>105153
*потом
>> No.105155 Reply
>>105153
> На счет всех точек между 1 и 2 сложно сказать однозначно, просто потому как фотонов не бесконечное число.
Мы можем взять масштаб достаточно большой, чтобы точек плоскости было существенно больше, чем фотонов. Уместно ли вообще говорить о движении?

Предположим, что вместо лазера мы взяли пушку, стреляющую ядрами в кирпичную стену раз в минуту. Предположим, что мы поворачиваем эту пушку. Тогда в минуту 1 в стене появилась дыра 1 в точке 1, минуту не происходило ничего, потом в стене появилась дыра 2 в точке 2 и так далее. Можно ли здесь вообще говорить о каком-то движении и тем более о скорости этого движения?
>> No.105156 Reply
>>105155
Ну так в том то и суть этого эксперимента. В случае описанном в статье аналогия примерно такая же. Информация не передается, фактически движения нет.
>> No.105158 Reply
>>105156
Журналисты назвали движением то, что не является движением?
То есть, по аналогии с пушкой, журналисты предположили, что на плоскости движется некая частица, которая раз в минуту вызывает появление дыр, и посчитали скорость движения этой частицы, взяв расстояние между дырами и поделив на время, и эта скорость оказалась очень высокой?
>> No.105159 Reply
>>105158
Не надо гнать на журналистов. В статье все описано нормально, в конце есть приписка о том, что это не сверхсветовое движение. Да и не очень понятно почему тебе так не очень нравится термин движение. Как еще назвать перемещение солнечного зайчика?
>> No.105160 Reply
>>105159
Потому что нет того, что можно было бы назвать зайчиком. Есть растущий набор дыр в кирпичной стене, которые никуда не движутся.
>> No.105333 Reply
File: 1.jpg
Jpg, 13.04 KB, 738×115 - Click the image to expand
edit Find source with google Find source with iqdb
1.jpg
Привет, доброанон. Помоги глупому школьнику решить задачу.
>> No.105334 Reply
>>105333
Пишешь два уравнения движения. Получаешь две прямые. Ищешь общий перпендикуляр. Навскидку.
>> No.105335 Reply
>>105333
Пишешь два уравнения движения. Получаешь две прямые. Ищешь общий перпендикуляр. Навскидку.
>> No.105340 Reply
File: 2.jpg
Jpg, 9.43 KB, 503×417 - Click the image to expand
edit Find source with google Find source with iqdb
2.jpg
ЧЯДНТ?
>> No.105344 Reply
Ну анон, ну помоги же.
>> No.105346 Reply
>>105340
1. Это не настолько быстрая борда.
2. Был неправ. Рассматриваешь относительно одной из точек. Т.е смотришь где расстояние между точкой и прямой, которую получишь сложением векторов, будет наименьшим.
>> No.105348 Reply
>>105346
Ты не мог бы это изобразить?
>> No.105351 Reply
File: 00000.png
Png, 19.22 KB, 839×475 - Click the image to expand
edit Find source with google Find source with iqdb
00000.png
>>105348
Бог пейнта врывается в тред.
>> No.105352 Reply
>>105351
Не уверен, но вроде бы понял. Спасибо за помощь.
>> No.106137 Reply
File: theory.svg
Svg, 0.01 KB, 744×1052 - Click the image to expand
theory.svg
Если свет - это электромагнитные волны, то они должны отклоняться в магнитных полях, верно?
Означает ли это возможность отклонения луча лазера способом как на пикрелейтеде? Если да, то почему никто этого не делает? Можно было бы сделать охуенный лазерный проектор без движущихся частей с отличным откликом. Или нужны сильные магнитные поля? Короче поясни баке по-хардкору.
>> No.106138 Reply
>>106137
Магнитное поле влияет на заряженные тела, а свет, грубо говоря, не является заряженным телом.
>> No.106139 Reply
>>106138
А, точно. Я понял. В кинескопах-то отклоняют пучок электронов, а они имеют заряд.
А если есть гипотетическое устройство для изменения гравитационного поля в двух направлениях, тогда можно будет отклонить луч в нём?
>> No.106140 Reply
>>106139
Карманная чёрная дыра, что ли?
>> No.106142 Reply
>>106140
Ящитаю использовать черные дыры в проекторах не практично.
>> No.106561 Reply
Вот скажите, господа диванные теоретики. Есть у нас электрон в суперпозиции спиновых состояний, <statee| = a <вверх| + b <вниз|. Мы меряем его спин некоторым прибором, который (вместе со всей окружающей вселенной) имеет состояние <stateuniverse|. Тогда полное состояние системы <statee|⊗<stateuniverse| = а <вверх|⊗<stateuniverse| + b <вниз|⊗<stateuniverse|. То есть, суперпозиция состояний с намерянным вверх и вниз спином. Эти состояния (со спином вверх и вниз) теперь эволюционируют независимо, так как уравнение Шредингера линейно, то есть для наблюдателя в одном из них как будто бы произошёл коллапс волновой функции.

Получилась, эффективно, многомировая интерпретация квантовой механики. Вопрос: какие проблемы в таком подходе? Вроде бы КТП не должно сильно испортить ситуацию.
>> No.106636 Reply
File: aNeXbJCAAd0.jpg
Jpg, 72.39 KB, 562×807 - Click the image to expand
edit Find source with google Find source with iqdb
aNeXbJCAAd0.jpg
Астроном-треда не нашел, по этому спрошу здесь.
Анон, что остается на месте взрыва сверхновой типа 1a?
>> No.106640 Reply
>>106636
Пульсар?
>> No.106642 Reply
>>106142
Скажи это моей зубной щетке с микроварп двигателем.
>> No.106651 Reply
http://habrahabr.ru/post/188020/
Анон, можно ли будет по этому способу создать лазерное оружие как в видеоиграх?
>> No.106654 Reply
>>106651
Да, если трактористы - женщины.
Блджад анон, ВНУТРИ кристалла. Выйдя из него, свет пойдёт на прежней скорости. Я уж молчу про остальные проблемы вроде зверского рассеивания энергии в воздухе.
>> No.106708 Reply
Сап, антуаны.
Хотел бы приобщиться к миру звезд.
Посоветуйте годной информации по физике маркообъектов, можно даже не до конца научной. я не против ознакомиться с недоказанными гипотезами

Т.е. материал должен в себе содержать инфу о всяких разных черных, серых, белых дырах, о физике звезд-гигантов, о теоретических сверхмассивных квазарах в центрах галактик и тому подобных фантастических и далеких объектах нашей Вселенной.
>> No.106709 Reply
>>95272
Анон, а если верхний рычаг упереть в потолок?
А к колесу электрогенератор?
Это ж чистый профит?
>> No.106710 Reply
File: perpetuum_mobile_...
Jpg, 41.12 KB, 469×384
edit Find source with google Find source with iqdb
perpetuum_mobile_1.jpg
File: usb_laptop_self-c...
Jpg, 39.29 KB, 530×350
edit Find source with google Find source with iqdb
usb_laptop_self-charger_cable.jpg
File: Boyle'sSelfFlowin...
Png, 59.27 KB, 320×282
edit Find source with google Find source with iqdb
Boyle'sSelfFlowingFlask.png
File: 1361978554_image2...
Png, 21.48 KB, 600×729
edit Find source with google Find source with iqdb
1361978554_image20.png

>>106709
А коль упрешь — не будет притока энергии от качания и вся установка остановится.
>> No.106711 Reply
>>106709
Установка остановится через некоторое время, если ее не колебать.
>> No.106713 Reply
>>106654
А можно сделать переносной луч Звезды смерти? Пока враждебные космические корабли бороздят космос, на орбите маленькой, но очень гордой планеты, висит Звезда смерти и испускает луч в маленький кристалл, который позднее будет взят на борт кораблями гордой страны.
>> No.106720 Reply
>>106713
С тебя Звезда смерти и кристалл.
>> No.106725 Reply
>>106710
> Boyle-sSelfFlowingFlask.png
А почему такая конструкция не будет работать? Вроде выглядит вполне логично.
>> No.106728 Reply
>>106725
А ты сам попробуй, увидишь.
>> No.106735 Reply
>>106725
Физика, 7 класс, закон сообщающихся сосудов.
>> No.106738 Reply
>>106735
Но жидкость вытекает из трубки, создавая там разрежение. Этим разрежением в трубку засасывается ещё жидкость, которая вытекает, и так далее. Хотя, скорее всего, вся жидкость просто перельётся обратно в чашку, а в область разрежения трубка соснёт воздух.
>> No.106739 Reply
>> No.106742 Reply
>>106739
Похоже, мы говорим об одном и том же, только разными словами. Ты ни разу не видел, как бензин из бака сливают в канистру? Или, может, ты в детстве не игрался с переливанием воды из высоко стоящего стаканчика в низко стоящий через трубку?
>> No.106750 Reply
>>106742
На пике разницы высот нет, так что не работает.
>> No.106983 Reply
File: flokdrow.png
Png, 281.50 KB, 801×480 - Click the image to expand
edit Find source with google Find source with iqdb
flokdrow.png
Анон, воспринимаемый человеком цвет зависит от частоты излучения или от длины волны?
>> No.106986 Reply
>>106983
Реально отказаться от гидродинамического представления поля в электродинамике? Есть ли учебники, оперирующие нечем иным, нежели потоками, циркуляциями и прочей «радостью»?
>> No.106987 Reply
File: bol1.pdf
Pdf, 0.27 KB, 421×595
bol1.pdf
File: kirillov-presenta...
Pdf, 0.08 KB, 842×595
kirillov-presentation.pdf

>>106986
Вот. Однако я не понимаю твоего негативного отношения к хорошей, годной, работающей системе абстракций.
>> No.107007 Reply
>>106725
Потери на трение! Если бы это была идельно текучая жидкость в сосуде с идеально невзаимодействующими стенками - то да, текло бы не останавливаясь. Просто по инерции, как летит в невесомости и вакууме невзаимодействующий ни с чем предмет.
>> No.107018 Reply
>>106739
Капиллярность, не?
>> No.107112 Reply
>>107018
ну допустим каппиляр. Допустим поднялась жидкость и смочила весь фитиль. А дальше у той жидкости, что на пике высоты есть два варианта стечь с двумя потенциальными ямами. Ну и потянется она в нижний сосуд, опустошит часть каппиляра, снова вернётся в него. И так до достижения равновесия системы.

Керосин вверх по фитилю идёт, потому что кончик фитиля осушается горением постоянно, по сути часть энергии горения тратится на подъём жидкости. Даже если в подобных системах впилить «диод» в виде односторонне-пропускающей мембраны или чего-то такого, потребуется внешняя энергия. Максимум что извлекли из подобных шуток — тепловой насос. То есть тепла излучается больше, чем тратится. Или наоборот тепло откачивается из области с температурой ниже в область с температурой выше. Ну ещё осмос по тому же принципу.

Если упрощать систему из зацикленного потребителья и генератора, прийдём сначала к маховику, а затем просто к неподвижному телу. То есть не важно будет движется оно или нет. Оно в идеале не отдаёт и не потребляет энергию (за счёт этого может хранить).

К сожалению даже версии зачёрпывания «из эфира» пока не подтвердились. Да что там эфир! У нас вон звезда есть, которой можно ни с кем не делиться, а поток её энергии до сих пор не освоен.
>> No.107157 Reply
Господа, не подскажете литературу по мазерам или другим квантовым генераторам?
>> No.107286 Reply
File: New-Canvas.jpg
Jpg, 18.37 KB, 536×475 - Click the image to expand
edit Find source with google Find source with iqdb
New-Canvas.jpg
Привет, физики.
Вот какая задачка любопытная (мне лично этот вопрос любопытен).

Есть цистерна с молоком. Молоко имеет свойство расслаиваться и верхний слой сливок более липкий. То есть если просто сливать в краник внизу, то налипнут на дне даже несмотря на то что оно наклонное. Ну может не молоко, другая расслаивающаяся жидкость с более липкой лёгкой фракцией.

Так вот первая идея, которая пришла мне в голову — гибкий шланг на поплавке (а), тогда первой будет сливаться верхушка. Но тогда остаток внизу не попадёт в шланг (поплавок опустится набок).

Первая мысль — сделать дырку внизу (в) у шланга, ощутимо меньшую чем его диаметр (б)

Что будет происходить в такой системе? Вверху давление жидкости меньше, из-за этого же жидкость потечёт только в нижнее отверстие и нужен либо клапан (срабатывающий от опусканий шланга на дно или понижения уровня) либо отдельный краник для слива остатков?

Понятно, что как бы проще тупо перемешивать постоянно жидкость или выкачивать насосом сверху (дырка шланга обращена вниз, поплавок выше). Но это энергозатраты.

В общем как бы ты решил подобную техническую задачу (верх слить первым, слить остатки как можно чище).
>> No.107292 Reply
>>106983
А стакан наполовину пустой или полный?
Частота и длина волны зависимы и нераздельно связаны.
>> No.107918 Reply
>>107157
Звелто, "принципы лазеров"
>> No.107925 Reply
>>106983
"Длина волны" = "скорость" / "частота". "Период" = 1/"Частота". Для электро-магнитных волн скорость в основном не меняется, так что между длиной волны и частотой, а так же периодом взаимно-однозначное соответствие. То есть, это вообще одно и то же по сути.

Кстати, если скорость непрерывно меняется, то термин длина волны имеет смысл?
>> No.107928 Reply
File: Untitled.png
Png, 61.75 KB, 1024×768 - Click the image to expand
edit Find source with google Find source with iqdb
Untitled.png
>>107286 Автоматический сифон?
>> No.108876 Reply
Доброе утро физики. репостну сюда
Я как гуманитарий долго и тщетно пытался разобраться в той части физики которая про переход материи в энергию и обратно на Википедии но не осилил.
Что думаете насчет статьи: http://dxdy.ru/topic36251.html
Если в ней есть хоть какой-то смысл, что может служить источником продольных волн в масштабах вселенной?
>> No.108879 Reply
>>108876
Доброе.

Утверждая возможные вопросы, заявлю, что я не считаю продуктивным разговор о большом взрыве, струнах, квантовой механике и искривлении пространства с людьми, которые не знают, что такое хаусдорфово топологическое пространство, некоторая окрестность каждой точки которого некоторым гомеоморфизмом отображается в R^n, и не могут доказать даже теорему Штейнера.

Рекомендую прочитать http://lesswrong.ru/w/Таинственные_ответы_на_таинственные_вопросы и ознакомиться с прочими статьями, выложенными на этом сайте.

Комментирую, раз уж ты просишь.
> еретические, неприличные вопросы
В физике не бывает еретических вопросов. Бывают вопросы, бывает бред.
> сказочных, необъяснимых сущностей вроде: суперструн, торсионных полей, темной энергии, черных дыр
Тёмной энергией называется то, что вынуждает Вселенную расширяться ускоренно. То, что Вселенная расширяется ускоренно, - самое значимое экспериментальное открытие девяностых, за которое в 2011 году дали нобелевку.
Существование объектов, похожих на прокурора чёрные дыры, доказано. http://lenta.ru/news/2013/04/02/blackandbrown/
Слово "сказочные" вызывает у меня неудовольствие.
> В противовес формальному, математическому
Нет, в сущности, никакой разницы между словом "шар" и словом "тензор". И то, и другое - геометрические объекты. Обывательское мнение, что обзывание некоторого материального образования шаром более просто, чем обзывание некоторого материального образования тензором, вызвано иллюзией понимания того, что такое шар. Я предполагаю, что если бы не школьное образование, обыватель не видел бы вообще никакой разницы между евклидовыми поверхностями, и не считал бы, что любой физический объект обязан быть геометрическим местом точек евклидова пространства.
Математика - язык физики, созданный во многом физиками специально для описания физических объектов. Некоторые математики даже называли математику частью физики. Автор статьи отказывается от специального физического языка в пользу журналистского языка. Это вызывает у меня неудовольствие.
> неограниченно долго удерживаемая в определенном объеме вакуума порция кинетической энергии
Отказ автора от математики немедленно приводит к абсурдным утверждениям. Энергия, вообще говоря, есть значение некоторого функционала. То есть, по современным представлениям, энергия абстрактна. Говорить о том, что в вакууме локализована энергия, это как говорить о том, что в вакууме локализовано число пять. Подобная вольность вызывает у меня неудовольствие.
> объеме вакуума
Автор, похоже, не знает о том, что абсолютного вакуума, в соответствии с предсказанным ruwiki://Эффект_Унру , нет, либо упускает это предсказание из виду. Это вызывает у меня неудовольствие.

Изложение автором видения квантовой механики вызывает у меня сильное неудовольствие, и я даже не буду комментировать, ибо получится tl.
> теоретический аналог материи, но и узнаем, как материя устроена на самом деле
Я бы хотел знать, что автор понимает под материей, потому что в современной физике понятие материи является дискуссионным. Лично я придерживаюсь точки зрения Ленина, который считал материю полным синонимом объективной реальности, и рассматриваю поэтому пространство-время как форму материи. По другой, более конкретной, точке зрения, материей является то, для чего можно зафигачить лагранжиан. Автор явно не придерживается ни одной из этих точек зрения, но понятием тем не менее пользуется, что вызывает у меня неудовольствие.

Рассуждения автора о насыщении среды энергией и о солитонах вызывают у меня недоумение, вопросы и неудовольствие.

Если бы автор потрудился построить математическую модель своей среды и если бы автор знал физику чуть глубже уровня википедии, в его рассуждениях можно было бы увидеть какой-то смысл. Мне это не удалось, и это вызвало у меня неудовольствие.

Вопрос о том, будут ли предложенные "солитоны" обладать качественными аналогами свойств элементарных частиц, невозможно решить без строгого определения солитона, построения математической модели и указания интерпретации известных фактов, допущу вольность речи, квантовой физики, в терминах будущей теории солитонов. Однако автор предлагает сперва построить теорию (как?) и только потом провести её математизацию. Я не вижу разумности, и это вызывает у меня неудовольствие.

Неясные термины вроде частицы среды, суперпозиция энергии и поля, создаваемые элементарными частицами, вызывают у меня неудовольствие.

После прочтения следующего абзаца у меня возникло ощущение, что я читаю щизофазический текст, что вызвало у меня неудовольствие.
> результаты, которые можно было объяснить только тем, что электроны не только чувствуют препятствия именно с одной или с двумя щелями еще на подходе к экрану, но и ощущают присутствие других электронов.
Нонсенс.
> Данная модель
Не является моделью вовсе, а похожа на поток рандомно выдранных из учебника физики терминов, произвольно пересыпанных туманными предикатами. Это вызывает у меня неудовольствие.
> позволяет получить ответы
Качество этих "ответов" вызывает у меня глубокое неудовольствие.

Заключение. Бессодержательный, семантически частично бессмысленный, частично противоречивый текст, вызвавший у меня неудовольствие.

Пойду спать.
>> No.108880 Reply
>>107928
Как вариант, но дело происходит в бочке. Так что для всплывающего над жидкостью стержня места нет.

>>108879
> В физике не бывает еретических вопросов. Бывают вопросы, бывает бред.
Даже так: бывают вопросы на которые есть смысл отвечать и бывают вопросы, на которые ответить прийдётся сделав слишком много допущений. И чтобы не превращать науку в религию с обилием толкований, последним стараются не заниматься.

Насчёт вакуума и энергии. Учитывая то, что масса — это и есть «свёрнутая» энергия… Занятно получается. С одной стороны «энергия в вакууме» это банально любой атом в своей кристаллической решётке. С другой, если полагать «много энергии без массы в одном месте без всего вообще», то это уже философсский вопрос совсем. Потому что если она там есть и строго замкнута, то мы никак не можем наблюдать (взаимодействовать), что она там есть.
> Эффект_Унру
Ухтышка! Не знал. Всё страньшеичудесатее относительнее и относительнее! Надеюсь, когда-нибудь наука перехлестнёт религию и философию, показав что всё строго субъективно и человеки смогут хуярить свою физику, пространства и время, с БлэкТеслой и Мэрилиинами Монро!
>> No.108883 Reply
>>108880
> Учитывая то, что масса — это и есть «свёрнутая» энергия
Энергия находится примерно в том же отношении с массой, в каком бутылка молока находится со своей ценой в рублях. Заявление звучит как "бутылка молока - это свёрнутые 50 рублей". Странно, то есть, звучит.
>> No.108884 Reply
>>108883
> Масса находится примерно в том же отношении с энергией, в каком бутылка молока находится со своей ценой в рублях
*фикс
>> No.108886 Reply
>>108883
E=mC^2 — по-моему вполне конкретное соотношение.
>> No.108889 Reply
>>108886
Никто не спорит. И масса так же эквивалентна энергии, как бутылка молока эквивалентна 50 рублям.
>> No.108892 Reply
>>108889
Таки бутылку можно продавать за сумму от нуля до бесконечности. А массу на энергию разменивать можно только строго по курсу. Это с одной стороны.
А с другой, если начинать разбирать материю на запчасти, там сплошные волны и энергетические уровни же.

Ну и последнее: всё это просто теория и модель. Наука соотносится с реальностью не более чем глобус с реальной планетой.
>> No.108907 Reply
>>108879
>>108880
Спасибо. Я так понял это >>108876 даже теорией назвать нельзя.
>> No.108908 Reply
>>108907
Не слушай дядек, а то совсем шарики за ролики заедут. Мы тут о своём, о философском.
Наверное это меня ты неправильно понял. Я излагал собственно принцим чистой науки без религии, то есть без упоротости и веры, что всё происходит именно так, как описывается, даже если всё описывается рабочей и практически применимой теорией и моделью. Модель всегда останется моделью, реальность — многогранной и выходящей за рамки каких-либо разумных возможностей пронаблюдать и измерить.

Из этого следует, что нельзя ни одну самую бредовую теорию заведомо считать бессмысленной и нельзя даже самую стройную рабочую модель считать непререкаемой истиной, иначе будет не наука а церковь. Данные, опорные точки и прочее принимаются не на веру (что так есть на самом деле) а просто как входные данные. То есть если задачу решают, то решают исходя из входящей информации и в каких-то рамках, а не решают вообще навсегда и для всего.

А дальше нужно проверять практически и теоретически параллельно. Не работает практически — нахуй не нужна. Работает и применима практически, но менее удобна, чем уже разработанная модель — не нужна. Объясняет больше, чем прежние, работает практически, удобнее в рассчётах — принимается как «новая выведенная формула» для той области, в которой хорошо работает. Это как например ньютоновской физикой пользуемся успешно, потому что в макромире для нас всякие квантовые эффекты и пространственновременные искажения несущественны или слишком усложняют задачу по сравнению с требуемой точностью определения.

Что касается перехода энергии в материю и обратно — это б тебе про термоядерный синтез и радиоактивный распад покурить, область хотя бы изученная. В этой теме главное не попутать материю (на деле-то непонятно, что такое материя вообще, но примем как массу, нечто, создающее взаимодействия) и состояние материи, то есть потенциальную энергию на макро, микро и квантовом уровне.


Что касается статьи, то я бы не стал её воспринимать, так как автор заведомо пытается выдать свою точку зрения или теорию за истину и практику, то есть пользуется религиозно-политическим а не научным подходом. Таким подходом нередко грешат и учёные мужи, создавая излишнее сопротивление новым теориям и укореняя старые, быть может не слишком удачные.

Что касается идеи автора «взглянуть системно и в целом» — это хороший подход.
Что касается фразочек «абсолютно подтверждённые» или «факт» или «реальность», то нет в науке на самом деле фактов, только теории и то, что принимается на данный момент как аксиома.

Любое наблюдение чего-либо даже не является наблюдением непосредственно этого самого. Мы не наблюдаем предмет, мы наблюдаем лишь свет, отражённый от него например, то есть у нас есть взаимодействие как минимум трёх сущностей (наблюдаемое, «щуп» — то чем щупаем, наблюдатель — мы). На деле же вот банально зрение: синий монохромный свет падает на лимон, мы видим почти чёрный лимон, то есть видим преимущественно свет, который обошёл лимон вокруг а не тот, который от него отразился. Всё это попадает в глаз, фокусируется, там своя физика и биохимия, а потом восприятие (многие легко узнают лимон, но описать особенности и даже выбрать цвет не в силах). И что мы можем сказать о лимоне? Ничего вообще толкового. Мы можем говорить лишь о том, как мы можем видеть лимон, какие эффекты он проявляет на нас. О полноценной реальности объекта (если она вообще существует) мы ничего сказать не можем.

Как-то мне приснился гиперреалистический момент. Я «увидел» дерево сразу во многих известных мне проявлениях (сезоны, движение, запах, разное освещение, разные ракурсы). Такое на всю жизнь впечаатется. И тем не менее это лишь узенькая щёлочка человеческого восприятия, через которую мы смотрим на мир. Поэтому я не поддерживаю автора и поддерживаю науку в мнении, что говорить о том, как «выглядит» реальность — это глупо и незрело.
>> No.108910 Reply
>>108908
> нельзя ни одну самую бредовую теорию заведомо считать бессмысленной
Можно. Теория "твоя помидора есть квантово-механический ротор дивергенции градиента потенциала синуса косинуса в фазовом пространстве конфигураций геодезической кривой над репером Чебышева-Эйлера комплексной связности кватерниона" бессмысленна.
>> No.108912 Reply
>>108908
> Что касается перехода энергии в материю и обратно — это б тебе про термоядерный синтез и радиоактивный распад покурить
Спасибо, лол понял что воспринимал науку именно как религию.
>> No.108915 Reply
>>108912
Цель игры «Табу» от Hasbro — помочь партнёру угадать слово, написанное на карточке, при этом не произнося ни этого слова, ни ещё пяти дополнительных слов, написанных на карточке. Например, тебе может понадобиться показать слово «бейсбол», не используя слов «спорт», «бита», «удар», «подача», «база» (и, разумеется, «бейсбол»).

Когда я узнал о существовании этой игры, я удивился. Почему бы просто не сказать «искусственный групповой конфликт, во время которого принято отбивать брошенный в тебя сферический объект при помощи длинного деревянного цилиндра, а затем перемещаться между четырьмя безопасными позициями»?

Элиезер Юдковский

Если ты не можешь с хорошим результатом играть в игру "Табу" с некоторыми понятиями, то ты не понимаешь этих понятий. Проверь себя. Возможно, твоё незнание даже глубже, чем ты сейчас осознал.
>> No.108917 Reply
>>108889
>>108892
Энергия и есть масса. Конкретнее - энергия, затраченная на образование системы (в системе отсчета где она покоится) будет её массой. А то, что меряют массу и энергию в разных единицах - историческое недоразумение.

Вообще, размерные константы не несут физического смысла. Будь то скорость света или постоянная Больцмана. Они просто означают, что люди ввели две единицы измерения там, где этого делать не нужно.
>> No.108918 Reply
>>108917
Дай определение энергии и дай определение массе.
>> No.108919 Reply
>>108910
> бессмысленна.
Осмысленность зависит от точки зрения. Вспомнил вот про близнецов—счётчиков из историй Оливера Сакса. У них была игра: сидели и называли друг другу числа, улыбались чему-то, соревновались. Для человека стороннего или не обладающего их способностями наблюдать пространство числе непосредственно, нативно, это абсолютно бессмысленно. Математик, посчитав, понял бы в чём особенность называемых чисел. А в чём их особенность и чем одно число изящнее другого — то ведомо только тем двум счётчикам, обладающим собственным восприятием.

Смысл создаём мы сами, называя бессмысленным то, что не укладывается в придуманное нами. Ещё вспомнился «психологический тест на маньяка» про логику «убить, чтобы ещё раз были похороны, чтобы ещё раз увидеть человека, который был увиден и понравился на похоронах».
>> No.108920 Reply
>>108918
Энергия - нулевая компонента вектора энергии-импульса, который определяет преобразование системы под действием группы Пуанкаре, бла, бла. Хуёвое определение, но лучшего с ходу не придумаю.

Масса - то, что определяет меру взаимодействия с гравитационным полем. И таки да, с некоторыми оговорками (вызванными кривизной моих определений, а не физикой) это одно и то же.
>> No.108921 Reply
>>108920
Ну вот же. Энергию ты назвал абстрактным математическим объектом, а массу - существующим объектом, который что-то определяет.
>> No.108924 Reply
>>108921
Они в одинаковой степени абстрактны, на самом деле, и энергию можно мерять без привлечения гравитационного взаимодействия. Суть в том, что гравитация действует на систему тем сильнее, чем больше энергия этой системы (то, что говорит нам ОТО). Поэтому энергию и массу разумно отождествить, чтобы не плодить лишних сущностей.
>> No.108926 Reply
>>108924
> и энергию можно мерять без привлечения гравитационного взаимодействия
Ух ты. Как?
>> No.108928 Reply
>>108926
А как её измеряли до появления ОТО? Кинетическая плюс потенциальная. Понятно, что абсолютное значение энергии ты так не получишь, но разницу увидеть можно. А еще можно смотреть на разные реакции с элементарными частицами, вот там эквивалентность массы и энергии видна во всех экспериментах.
>> No.108930 Reply
>>108928
> А как её измеряли до появления ОТО?
А я не знаю, лол. Это для меня какое-то шаманство. Берут тело и говорят: квадрат его скорость помножить на его массу пополам будет энергией. Что это такое, откуда это берётся, что сохраняется, почему должно сохраняться - не говорят.
>> No.108931 Reply
>>108930
В рамках Ньютоновой механики (а если быть более точным - Лагранжевой) энергией называется величина, сохраняющаяся из-за однородности времени (нет разницы, происходит всё сейчас или "10 минут назад", например), а конкретный её вид вытекает из экспериментальных результатов. Если интересно, можешь самое начало Ландау-Лифшица первого почитать. Там это вводится аксиоматически, конечно, но подробно и более-менее понятно.
>> No.108934 Reply
>>108930
x - перемещение, v - скорость, a - ускорение, F - сила, m - масса.
dk - изменение k.

По определению v = dx/dt.
vdt = dx
dx = vdt

По определению a = dv/dt.

По второму закону Ньютона ma = F.
ma * dx = F * dx
mdv/dt * dx = F * dx
mdv/dt * vdt = F * dx
mvdv = F * dx
mvdv = (mv^2/2)
т.к. d(mv^2/2) = d(v^2 * (m/2)) = 
 = (m/2) * d(v^2) = (m/2) * d(v^2) = (m/2) * 2v dv = mv dv

Итак,
(mv^2/2) = F * dx

Пусть есть система тел, на которую не действуют внешние тела.
Равнодействующая системы равна 0.
(mv^2/2) = R * dx = 0

Но дифференциал равен нулю тогда и только тогда, когда под ним константа.
mv^2/2 = const
Провозгласим mv^2/2 кинетической энергией.
>> No.108936 Reply
>>108934
Блин.
> mvdv = (mv^2/2)
mvdv = d(mv^2/2), конечно же.
> (mv^2/2) = F * dx
d(mv^2/2) = F * dx
> mdv/dt * dx = F * dx
Здесь мы просто от балды и желания теоретизировать скалярно умножаем второй закон на какое-то dx.
>> No.109442 Reply
File: 4bf456d67720c9f0f0323d470550c36a.jpg
Jpg, 19.54 KB, 200×273 - Click the image to expand
edit Find source with google Find source with iqdb
4bf456d67720c9f0f0323d470550c36a.jpg
Извините, что врываюсь в тред, но пикрелейтед годный учебник физики для школьника, проебавшего уроки физики? Я в 11 классе. [spoiler]в физике полный нуль[/spoiler]
>> No.109477 Reply
>>109442
Школьными учебниками можно только подтереться, если хочешь познакомиться с физикой на приемлемом уровне - Феймановские лекции по физике.
>> No.109479 Reply
Зельдович-Я.Б.,-Яглом-И.М.-Высшая-математика-для-н.pdf
>>109442
Прочитай от корки до корки.
>> No.109604 Reply
>>109477
Поясни, анон.
> Школьными учебниками можно только подтереться
Из-за скудности материала, или из-за неправильного толкования?
>> No.109607 Reply
>>109604
Скажу за анона. Школьные учебники не только скудно подают материал, но и написаны нередко людьми не особо профессиональными в этой области и не особо заинтересованными, плюс держащими школьника за слабоумного. Когда детей взрослые с рождения держат за слабоумных, такими они и вырастают. Пруф были в виде опыта с двумя классами. В одном сказали учителю, что дети слабые и отстающие, в другом, что особо одарённые. Дети были совершенно рэндомные, успехи во втором классе выше средних, в первом — ниже. Отношение передаётся.

Профессионалы, которые пишут книжку а то и не пишут (а за ними записали из-за ценности знаний и подачи) — другая каста. Они могут увлечь, заразить любовью к предмету, а так же подать не просто информацию, но понимание процесса, привить мышление.

Школа в основном занимается тем что «даёт знания». Знания сами по себе — штука не только бесполезная но и вредная, голову забивать. Ну знал я раньше столицы стран, теперь забыл. А толку-то? Даже если я знаю в каком направлении страна находится, какой в этом смысл? Никакого. Мусор в голове и повод для хвастовства бабушек эрудированным чадом «а у наше-го то больше говна в голове чем у вашего!»

И сравни уровень преподов в ВУЗе или школьных. Разные касты, разное понимание, хотя и в ВУЗе есть такие, которые лекции толдычат по писанному.

Фейнман же — человек упоротый по теме физики, увлечённый да и довольно весёлый вообще.

Быть может есть и школьные хорошие, которые автор писал от отчаяния, что шлаком детей пичкают, а не ради гонорара и от безделия. Но ВУЗовский уровень более глубже смотрит на вещи. В итоге избавляет тебя от надобности выкидывать старые модели и вникать в новые, взамен давая возможность начать с более глубоких моделей а более простые воспринимать как частное упрощение.

К примеру нас на химии дрочили с периодами, массами и прочим для элементов. Потом учили запоминать ряд оболочек. И только в конце объяснили принцип, почему электронная оболочка строится именно так. То есть оказалось, что заучивать совершенно не нужно было, её можно вывести, выстроить, зная три-четыре «особых» пункта (электронные провалы, перескоки электрона на другой энергетический уровень). А потом оказалось, что там никакая не форма оболочки, а просто математическая абстракция, удобная для описания свойств волны. Сначала учат, что электрон вращается, потом что «размазан» по оболочке а потом только, что есть некоторая вероятность обнаружения электрона в определённой точке и сумма всех вероятностей даёт «поверхность» которая и называется электронной оболочкой.


Мне бы было проще, если бы начали с конца, приведя к началу и упрощению.
>> No.109608 Reply
>>109607
> а потом только, что есть некоторая вероятность обнаружения электрона в определённой точке и сумма всех вероятностей даёт «поверхность»
А ведь потом заявляют, что это всего лишь одна из интерпретаций, и что, в общем, непонятно, как эта шайтан-абстракция работает.
>> No.109609 Reply
>>109608
Ага. И говорить детям о том, что всё — лишь абстракция и модель, которая просто позволяет делать практические предположения но не даёт ответа на то, как же на самом деле устроено, нужно в начале каждого научного или религиозного курса.
>> No.109612 Reply
>>109609
И что делать, когда умный ребятёнок спросит "а что такое на самом деле"?
>> No.109613 Reply
File: Политрук.jpg
Jpg, 467.22 KB, 524×1476 - Click the image to expand
edit Find source with google Find source with iqdb
Политрук.jpg
>>109612
Заставить покаяться неверного.
>> No.109614 Reply
>>109612
искать умного взрослого, который ответит:
«так как есть, в полном объёме свойств, независимо от наших представлений»
>> No.109736 Reply
>>88754
Бамп как всегда, годному треду.
И сразу же вопрос: антуаны, какой справочник лучше использовать для абитуриента школьника-меня? Готовлюсь к ЕГЭ с Ландсбергом. Теперь бы и справочник не помешал бы.
>> No.109796 Reply
File: картинка1.JPG
Jpg, 33.88 KB, 915×217
edit Find source with google Find source with iqdb
картинка1.JPG
File: картинка2.JPG
Jpg, 74.12 KB, 1136×203
edit Find source with google Find source with iqdb
картинка2.JPG

Няши, помогите решить, пожалуйста.
>> No.109805 Reply
>>109796
Считать IV-III и II-I как два единых блока, понять, что ускорение у них общее, и написать второй закон Ньютона?
>> No.109809 Reply
>>109796
F=ma
где a для всех общее, а m — сумма масс, известная.

Дальше известно, что нашу пружинку с силой 1 Ньютонов в одну сторону тянет движущая сила, а в другую равно так же — инерция (сила действия равна силе противодействия же).
5 Ньютонов в одну сторону да пять в другую. То есть 2 килограмма при неизвестном ускорении создают инерцию в 5 ньютонов.

Сударь, извольте открыть законы Ньютона и включить голову. Я 15 лет назад в школе учился и мне эти ньютоны в жизни не нужны были ни разу, однако даже с моей точки зрения очевидно, что задача элементарна а вы, мой юный друг, лентяй охуевший, поленившийся даже задачу перепечатать нормально текстом.
>> No.109810 Reply
>>109809
> 1 Ньютонов в одну сторону
5 Ньютонов несчастных тянут пружину в каждую из сторон, ну или 10 в одну, это от системы отсчёта уж.
>> No.109815 Reply
>>109805
>>109809
Хорошо. И каков же численный ответ, няши?
>> No.109834 Reply
Привет, анон. Дело в том что я лентяй и не учил физику, то есть у меня есть только самые базовые понятия о ней. Но мне внезапно интересна квантовая физика, так как она может дать ответ на некоторые интересные вопросы. Суть вопроса в том есть ли смысл читать книжки по квантовой физике без серьезного знания обычной? Или я без этого ничего не пойму? Если да, то что можно почитать для начала?
>> No.109835 Reply
>>109834
> есть ли смысл читать книжки по квантовой физике без серьезного знания обычной
Нет.
> что можно почитать для начала
Гельфанд-Шень, алгебра.
Ландсберг, элементарный учебник физики.
Зельдович-Яглом, математика для начинающих физиков и техников.
Фейнмановские лекции по физике.
>> No.109859 Reply
File: dreamland___above_than_clouds_by_anuk-d2f04rh.jpg
Jpg, 444.84 KB, 1600×1221 - Click the image to expand
edit Find source with google Find source with iqdb
dreamland___above_than_clouds_by_anuk-d2f04rh.jpg
Анон, привет. У меня возник такой вопрос, больше от нечего делать, чем осознанный. Что было бы, если бы классическая физика была верна? Ну, в каком-то смысле, конечно, ничего бы не было, потому что электроны бы наебнулись на ядро хотя бы, однако что было бы, если бы физика не столкнулась с большим количеством проблем в конце девятнадцатого столетия? Ультрафиолетовая катастрофа не произошла бы, опыт Майкельсона-Морли показал бы, что свет меняет свою скорость... Как выглядел бы современный мир, что ты думаешь, анон?
>> No.109868 Reply
>>109815
Иди учись, лентяй.

>>109834
Вопрос не только в понимании «обычной» но и в развитии мышления, понимании общих принципов.

>>109859
Наука в принципе не может быть верна или неверна. Она оперирует пригодной для решения задач моделью а не реальностью. То есть как не знали мы что на самом деле происходит, так и не знаем. У нас есть чёрный ящик. Мы знаем, что с высокой вероятностью при подаче в него сигнала А, получим в ответ Б. Что в ящике мы никогда не узнаем.
>> No.109871 Reply
>>109868
Пожалуйста, скажи ответ.
>> No.109875 Reply
>>109871
Она верна.
Ничего бы не было, или так что никто не в силах представить.
другой анон
>> No.109878 Reply
>> No.109879 Reply
>>109868
Ты софист без фантазии.
>> No.109881 Reply
>>109809
Ну что же вы, няши. Дорешайте задачку до конца, пожалуйста. Мне всего лишь нужно проверить себя.
>> No.110318 Reply
>>109736
Анон, я все-таки на тебя рассчитываю.
>> No.110393 Reply
Привет, Анон!
Скажи, можно ли говорить что электрическое поле одно и существует в каждой точке пространства, или же наоборот, электрических полей много и они чудесным образом рождаются вблизи электрически заряженных тел?
>> No.110394 Reply
>>110393
Дай подкорректирую. Щито такое поле , с чем это едят , как оно там без наблюдателя то бедненькое ?
>> No.110395 Reply
File: 1381602966006.png
Png, 1.09 KB, 300×20 - Click the image to expand
edit Find source with google Find source with iqdb
1381602966006.png
>> No.110396 Reply
File: 1381603957547.png
Png, 1.22 KB, 300×20 - Click the image to expand
edit Find source with google Find source with iqdb
1381603957547.png
>> No.110409 Reply
>>110393
Одно
>> No.110423 Reply
Суп, доброчан. У меня в этом семестре курсовая по тензорному исчислению, так вышло, что интересные мне темы уже разобрали, подскажи что-то похожее. Интересны именно физические и механические приложения.
1. Обоснование классификации конечных кристаллов по группам симметрии
2. Определение числа независимых компонент тензоров линейных физиче-ских свойств конечных кристаллов различной симметрии
18. Исследование тензора Римана-Кристоффеля
19. Теорема Остроградского-Гаусса для векторного и тензорного полей, ее следствия
20. Теорема Стокса для векторного и тензорного полей, ее следствия
21. Определение градиента, ротора, дивергенции векторного и тензорного оплей в различных криволинейных системах координат.
23. Определение тензора инерции, его свойства, значение тензора инерции для различных геометрических объектов (куб, балка, эллипсоид, кольцо, цилиндр и т.д.)
>> No.110428 Reply
Собрался сдавать физику(ЕГЭ), что лучше выбрать для подготовки,Ландсберга, или это http://mathus.ru/phys/book.pdf
>> No.110436 Reply
>>110428
И то, и то.
>> No.110570 Reply
File: 1381790290117.png
Png, 50.76 KB, 500×333 - Click the image to expand
edit Find source with google Find source with iqdb
1381790290117.png
Любые два квадрата равномошны, в них одинаковое количество точек, даже если они разного размера. Картинка (иллюстрация к закону обратных квадратов) предполагает, что силовых линий конечное число, так ведь?
>> No.110614 Reply
Господа, поясните пожалуйста про задачу: "С какой скоростью должен бежать Иисус по озеру, чтобы не повалиться под воду?"
>> No.110616 Reply
>>110614
Он вообще стоять может, он же Иисус.
>> No.110629 Reply
>>110614
Сильно зависит от того что вы проходите в школе.
Если закон притяжения масс, то это первая космическая(твой Иисус попросту летит по орбите но очень низко).
>> No.110631 Reply
>>110629
На такой скорости трение о воздух адовое же. Сгорит же.
>> No.110634 Reply
>>110629
Скорее имеется ввиду поверхностное натяжение воды.
мимо
>> No.110637 Reply
>>110629
1) я уже не в школе, первокурсота-кун
2) просто любопытна задача, хотелось бы в буквах увидеть вывод формулы скорости через любые другие данные,будь то площадь стопы или вес Иисуса.
>>110631
А вот значит с небес на землю и обратно он может нормально, а у поверхности земли сгорает?
>> No.110643 Reply
>>110614
Гугель говорит, что 20-30 метров в секунду. Но у человека при его КПД разогрев самих мышц от такой скорости будет вполне достаточен, чтобы его обуглить и испарить воду.
>> No.110644 Reply
File: 12723834135037.png
Png, 199.85 KB, 544×400 - Click the image to expand
edit Find source with google Find source with iqdb
12723834135037.png
>>110643
> человека при его КПД
А какой у человека КПД?
>> No.110645 Reply
>>110644
Не имею понятия, но судя по тому, что заявленная мощность на поддержание такой скорости - 12 киловатт, а выделяемое при этом тепло 120 - не айс.
>> No.110737 Reply
>>110643
Да мне не цифры нужны, хотябы с моделью помогите, как представить Иисуса? Ввиде бруска на воде?, где площадь соприкосновения с водой есть сумма площадей стоп?
>> No.110741 Reply
>>110737
Он ведь касается воды только 1 ступней
>> No.110744 Reply
>>110737
Но ведь нужно ещё учитывать и сопротивление воздуха, потому что на большой скорости оно будет значительным.
>> No.110745 Reply
>>110744
Воздух стоит учитывать только как необходимость постоянно действовать на тело силой, такой же по модулю, как и сила трения, чтобы поддерживать постоянной кинетическую энергию тела, которая тратится на разогрев вследствие трения. Значение скорости в первоначальном вопросе не зависит от трения воздуха. Если так хочется учитывать воздух, то каким - нибудь боком прикрутите гидродинамику и закон Бернулли, например.
>> No.110749 Reply
>>110737
YouTube: Jesus Christ Lizard
Видели же, кстати?
>> No.110767 Reply
File: zdFBzLlt24M.jpg
Jpg, 357.15 KB, 1280×1706 - Click the image to expand
edit Find source with google Find source with iqdb
zdFBzLlt24M.jpg
>>88754
Может я не по теме треда, но гугл мне не помог. Аноны, не лежит ли у вас где-нибудь в электронном виде задачник: Л. А. Кирик "Задачи по физике для профильной школы с примерами решений."? Очень нужно.
>> No.110840 Reply
>> No.110855 Reply
File: 1292657734249.jpg
Jpg, 45.69 KB, 200×300 - Click the image to expand
edit Find source with google Find source with iqdb
1292657734249.jpg
>>110840
Унеси это обратно, пожалуйста.
>> No.110878 Reply
Здесь случаем не водится хардкорных физиков, которым можно задавать глупые но предметные вопросы про суперсимметрию и подобное?
>> No.110906 Reply
>> No.110908 Reply
>>110906
Водятся.
>> No.110926 Reply
>>110908
Отлично! Значит, суперсимметрия, как я понимаю, есть некоторое расширение обычной Пуанкаре. То есть это пространственно-временная симметрия. Она допускает описание в терминах (вероятно, расширенной) структуры пространства, как группа Пуанкаре? И почему говорят, что теория с суперсимметрией обязательно содержит гравитацию - казалось бы, мы и без неё требованием локальности симметрий с генератором P поимели бы то же самое, а суперсимметрия только добавляет метрическому полю суперпартнёра.
>> No.111039 Reply
Ну что вы, теоретики?
>> No.111090 Reply
>>110926
Ну ты бы почитал про симметрию для начала бы. Грубо говоря, генераторы для групп\алгебр связываются либо антикоммутационными соотношениями (в квантовой физике), либо коммутационными соотношениями (алгебра Ли, узко если, то пространственные генераторы для группы Пуанкаре). Суперсимметрия - это такая вот магическая штука, которая объединяет два типа генераторов. Дальше сам продолжишь?
>> No.111091 Reply
>>111090
> антикоммутационными соотношениями (в квантовой физике)
Я имею в виду в первую очередь матрицы Дирака и другие n-мерные фермионные аналоги. Вообще, няша, копай в генераторы суперсимметричных алгебр и в генератор алгебры Пуанкаре. Посмотри на них внимательно. Генераторы алгебры Пуанкаре отвечают за метрику. Условно, к ним можно прикрутить связности - получится что-то похожее на гравитацию, но пока без тензора энергии-импульса, то есть без вещества. Топологическая гравитация, если хочешь. По этой причине говорят, что суперсимметрия (расширение вот этой вот алгебры) содержит гравитацию. Просто тебя никто не учил писать гравитацию через группы\алгебры, как квантовую механику. (И тому имелась очень важная причина: гравитацию можно написать внешне похожей на расслоение над многообразием, но по факту это совсем другой сюжет. То есть, например, электромагнетизм и уравнения Максвелла переписываются как формы над многообразием, где вектор А (поправьте меня тут) и есть связность).

Суперсимметричная алгебра содержит и генераторы для разных квантовых моделей, но тут надо аккуратно смотреть на алгебры Ли. Они все очень похожи, если над реальными числами их брать. В них легко запутаться, так что если я начну объяснять (а у меня это плохо получается, как ты уже заметил), то будет еще хуже.
>> No.111095 Reply
>>111090
>>111091
Матрицы Дирака возникают когда мы строим представления группы SO(1,3) мне казалось, квантовая механика там не очень нужна. Вообще все разговоры про симметрии, что пространственные что калибровочные - они классические. А в квантовой механике генераторы групп симметрий разве по-другому себя ведут и связываются антикоммутационными соотношениями? Не припоминаю такого.

А вопрос мой был вот о чём: нельзя ли наглядно представить суперсимметрию как генераторы группы преобразований какого-нибудь объекта? Условно говоря, взять вместо пространства Минковского две его копии, или какую-нибудь хитрую локальную структуру или ещё что. Разглядывание соотношений на генераторы ситуацию не проясняет. Понятно, что из-за неправильных коммутационных соотношений с этим будут проблемы, но хоть какой-то физический смысл суперсимметричным преобразованиям можно придать?
>> No.111096 Reply
>>111095
Во-первых, ты неправильно разграничиваешь классику и квантовую теорию. Квантовая теория - это не ТОЛЬКО КТП, ТОЛЬКО МАТРИЦЫ РАССЕЯНИЯ, ТОЛЬКО ХАРДКОР. Как только мы начинаем писать соотношения над волновыми функциям (векторами в С^n), мы сразу же начинаем писать квантовую модель. Поэтому:
> Вообще все разговоры про симметрии, что пространственные что калибровочные - они классические.
Все эти разговоры, в основном, строятся около стандартной модели и появления массы в результате нарушения спонтанной симметрии. Это относительно новая, разработанная в 60х годах модель. А квантовая механика, описанная в Ландафшице - это 30е годы. Суперсимметрия ближе к 70-80 строилась, так ведь?
> Матрицы Дирака возникают когда мы строим представления группы SO(1,3) мне казалось, квантовая механика там не очень нужна.
Ну это вообще же пушка. Матрицы дирака - это частный случай алгебры Клиффорда. Алгебра Клиффорда описывает фермионы. Для фермионов всегда пишутся антикоммутационные соотношения. Фермионы - квантовые объекты. Ок?
> А вопрос мой был вот о чём: нельзя ли наглядно представить суперсимметрию как генераторы группы преобразований какого-нибудь объекта?
Можно. Но так делают только струнщики, потому что они дальше развивают эту модель. Но если честно, я могу сказать только общую идею, как бы я на это смотрел. У тебя есть физический мир и частички двух типов: фермионы и бозоны с внутренними степенями свободы. То есть физический мир - это многообразие с расслоением. Ты знаешь, что мир не зависит от того, как ты на него смотришь. Значит, Лагранжианы в модели должны быть симметричны относительно какой-то группы преобразований. Перенос системы отсчета порождает группу (я везде буду говорю алгебру, потому что с алгеброй проще работать):: алгебру Пуанкаре (над пространстом действительных чисел). Симметрии во внутренних (комплексных!) пространствах задаются коммутационными (для бозонов) и антикоммутационными соотношениям (для фермионов).

Пауза. Контрольные вопросы:
1) Что такое связности в гравитации и в теории Янга-Миллса.
2) Как записаны лагранжианы в стандартной модели: над полем R или над полем С? Как ввести туда гравитацию (простейший ответ: ввести метрический тензор и добавить ..что??..)

То есть, блин, уже даже на таком минимальном языке возникает куча вопросов, да? У нас есть расслоенное пространством, мы изучаем его симметрии. И у нас, блжд, проблема. Локально можно написать всю теорию в калибровочном варианте: и гравитацию, и стандартную модель. Но глобально так не получается (гравитация не калибруется, хнык). Чтобы это сделать глобально, нужно представить, что наблюдаемый мир - это часть гораздо большего пространства. Вообще-то, струнного\бранного\... пространства. Но суперсимметрия говорила (тут нужен эксперт, в этом я не уверен), что пространство можно расширить до суперсиммтричного и рассматривать преобразования над ним. Это похоже на физический смысл?
>> No.111121 Reply
>>111096
> Все эти разговоры, в основном, строятся около стандартной модели и появления массы в результате нарушения спонтанной симметрии.
В том варианте, который я слышал, эта песня поётся примерно так: вот, есть группа Пуанкаре, и каким-либо претендующим на элементарность физическим объектам (читай - частицам) должно соответствовать неприводимое представление этой группы. Дальше мы классифицируем эти представления и обнаруживаем, что частице соответствуют масса и спин, в соответствии с полученными представлениями. Ну или спиральность. Разговоров про кванты и про природу частиц на этом этапе нет вообще.
> Матрицы дирака - это частный случай алгебры Клиффорда. Алгебра Клиффорда описывает фермионы. Для фермионов всегда пишутся антикоммутационные соотношения. Фермионы - квантовые объекты. Ок?
Эм, окей. Но было бы очень мило, если бы ты меня послал туда, где можно почитать про теорему о связи спина и статистики, и желательно ещё о физическом смысле всего этого безобразия. В частности, можно ли придать хоть какой-то смысл фермионным полям в классическом пределе и их антикоммутации.
> Пауза. Контрольные вопросы:
Это мне или просто так? Связность в гравитации - ну, связность и есть, Г с тремя индексами. В теории ЯМ - калибровочное поле, то бишь формы A. Вообще связность по-моему определяется как поле подпространств размерности базы в касательном расслоении твоего расслоения, которое показывает "нулевое направление" при движении вдоль базы, ну и как-то там самосогласованное. А формы А - это просто набор 1-форм в касательном расслоении расслоения, которые задают это подпространство обнуляясь на нём. Но тут я могу безбожно врать
Лагранжиан пишут над полем R, ибо действие действительно (лол). Поля могут быть комплексными, но итоговое выражение - синглет под U(1). Для гравитации нужно добавить метрику, добавить корень из скалярной кривизны отдельным слагаемым, и меру интегрирования ещё пофиксить, то бишь на корень из минус определителя метрики помножить.
> Но суперсимметрия говорила (тут нужен эксперт, в этом я не уверен), что пространство можно расширить до суперсиммтричного и рассматривать преобразования над ним. Это похоже на физический смысл?
Очень похоже! Я и хочу представить суперсимметричное пространство, или найти где-нибудь его описания.
>> No.111122 Reply
>>111121
> набор 1-форм в касательном расслоении расслоения
1-форм в самом расслоении, конечно же
selffix
>> No.111127 Reply
>>111121
Конкретно и развернуто ответить не смогу сейчас. Но у тебя какое-то формальное видение объектов, мне кажется. Могу дать только общий совет: читай введение к (около)струнным статьям на архиве. Там посоны на 2-3 страничках обычно поясняют физический смысл моделей. Собственно, я в свое время про суперсимметрии читал из какого-то обзора оттуда. Вот такого типа сюжеты смотри: http://arxiv.org/abs/0712.0689

Может быть, на общем уровне теорфиз ты знаешь лучше меня. Все же я давно ничего дальше своей локальной чисто математической области не читал. Я в свое время просто переключился с учебников на чтение абстрактов и введений к статьям. И это дало мне гораздо больше, чем универ, научник и учебники. Добра!
>> No.111146 Reply
>>111127
Ух ты, как раз искал не сильно заморочистое введение в AdS/CFT. Спасибо большое!
Setting sun diagram, это ж надо. Японцы такие японцы.
>> No.111446 Reply
Посоветуйте что-нибудь по электромагнетизму, с нуля нулей.
>> No.111449 Reply
>>111446
Сивухин, «Электричество и магнетизм».
>> No.111471 Reply
Почему угловая скорость определяется именно так, как она определяется?
>> No.111473 Reply
>>111471
Что именно тебя смущает? Направление?
>> No.111474 Reply
>>111473
Да.
>> No.111475 Reply
>>111474
Меня это тоже смущает. Но лучшего способа задать направление вращения через векторы так и не придумали.
>> No.111536 Reply
>>111475
А кто первым до этого додумался?
>> No.111538 Reply
>>111536
Понятия не имею. Но векторные произведения ввёл вроде бы Гамильтон (они пошли из операций над квартернионами), так что скорее всего пошло от него.
>> No.111645 Reply
>>110570
А как тебе теорема Марцинкевича о возможности непрерывно отобразить отрезок в квадрат?
>> No.111647 Reply
>>111645
А при чём здесь это? Картинка ссылается на густоту силовых линий. Её автор говорит, что через более крупный квадрат проходит меньше силовых линий, чем через более мелкий.
>> No.111650 Reply
>>111647
Я, конечно, не физик, но разве можно говорить о числе силовых линий? Разве это не абстракция?
>> No.111653 Reply
>>111647
Поток это \int E·dS. Поток постоянен, площадь [замкнутой] поверхности увеличилась пропорционально квадрату, значит, сила стрелочек во столько же ослабла. Количество стрелочек считать бессмысленно, их всё равно столько же, сколько точек.
>> No.111654 Reply
>>111645
Она теорема Мазуркевича, блджад.
>> No.111763 Reply
Прочитал первую главу Фейнмана из тома "Механика". Смог решить только 2 задачи из предложенных, я один такое быдло?
>> No.111765 Reply
>>111763
Там вообще нет задач.
>> No.111769 Reply
>>111765
Есть дополнительный том с задачами.
>>111763
Другой учебник попробуй. Может, просто не твое.
>> No.111773 Reply
>>111769
Какой например? Ландау - для старших курсов ВУЗа; Мякишев - все говорят хуита; Фейнман - мне ни идет; Остается Касьянов 10 класс, может потом после него Фейнмана?
Я школьник, не учивший физику, даже не ориентируюсь, могу только решать задачи в школе по формулам.
>> No.111780 Reply
>>111773
Сивухин, Савельев, Иродов, в конце-концов.
>> No.111973 Reply
File: оптическая-длина-пути.png
Png, 13.07 KB, 819×460 - Click the image to expand
edit Find source with google Find source with iqdb
оптическая-длина-пути.png
Физикач, помоги разобраться с принципом наименьшей оптической длины пути. Суть вопроса на пикче.
>> No.111974 Reply
>>111973
Отбой, разобрался. Пусть путь не минимальный, он всё равно экстремальный, пусть это и локальный экстремум.
>> No.111977 Reply
>>111773
Ландау не просто для старших курсов ВУЗа, он именно по теоретической физике - абсолютно иной метод подачи материала и необходим гораздо больший математический аппарат. Это так, для справки, чтобы не попытался ткнуться в него, когда захочешь термодинамику подучить.
>> No.112298 Reply
Знаю, уже было, но всё же спрошу.
Посоветуйте хороший, годный (хорошо написанный, etc) учебник/лекции по физике. Вовсе не обязательно отечественный, можно на английском языке, главное- годнота.
>> No.112306 Reply
>>111780
А что-нибудь кроме них есть?
>> No.112310 Reply
>> No.112409 Reply
Привет физик-кун. Поясни пожалуйста одно явление. В школе учат, что приливы и отливы в мировом океане бывают из-за влияния гравитационного поля луны. А от солнца никаких микроприливов не бывает? Или может время наступления искажается с поправкой на суперпозицию других гравитационных полей?
>> No.112424 Reply
>>112409
Солнце меняет интенсивность прилива. Если оно расположено так же, как Луна, прилив сильнее, если под углом - прилив слабее.
>> No.112539 Reply
File: andromeda-2.jpg
Jpg, 152.22 KB, 756×582 - Click the image to expand
edit Find source with google Find source with iqdb
andromeda-2.jpg
Многоопытный анон, к тебе вопрос. На твой взгляд, с педагогических позиций (подавание материала), какой учебник лучше: Савельев, Сивухин или Иродов? И почему?
>> No.112575 Reply
>>112539
Смотря что ты хочешь учить и каких высот желаешь добиться.
Так, например, оптику Сивухина с трудом читают даже специалисты в области оптики.
Поначалу приятно читать Фейнмана - доступным языком и жизненными примерами, как для самых маленьких. Но - мало математической строгости.
>> No.112578 Reply
>>112575
Математическая строгость у Фейнмана и, скажем, в Ландавшице одинаковы. Ни там, ни там ни капли приличных обоснований. Так что это не страшно.
мимоматематик
>> No.112581 Reply
>>112578
Математической строгости в физике не везде можно найти. Скажем, лет 50 уже повсеместно пользуются в вычислениях Фейнмановскими интегралами по траекториям, которые с математической точки зрения не определены вообще.
>> No.112644 Reply
>>112575
> Смотря что ты хочешь учить и каких высот желаешь добиться.
Для понимания вузовского курса физики для химика. Можно сказать, всего по немногу. Чтобы и проработка была глубже школьной, и глубина была недостаточной для студента-физика.
>> No.112650 Reply
>> No.112664 Reply
>>112650
Спасибо.
>> No.112758 Reply
Привет, добранон.
Объясни мне один момент из квантовой физики. Я (студентота) разбираю задачку, и в ней орбитальное квантовое число (которое определяет максимальное значение проекции момента импульса на некоторую выбранную нами ось) оказалось равно нулю. Но ведь это означает, что тем самым мы определили проекцию момента импульса на каждую ось, и она оказалась равной нулю! А это входит в противоречие с соотношением неопределённостей.

Прошу, спаси мой мозг от самоподжарки, объясни, в чём суть, или скажи, что это авторы задачника обдолбались.
>> No.112759 Reply
>>88754
Привет анон физик. Есть один вопрос, заранее извиняюсь если что то не правильно понял.
Вопрос теоретический из разряда научной фантастики(наверное).
Вот собственно он: Если учитывать что вселенная расширяется то следственно и все в ней расширяется соразмерно, а если моя кошка :3 переместится в прошлое то не будет ли она великаном ?
>> No.112777 Reply
>>112759
Нет, вселенная расширяется в том смысле, что астрономические объекты разбегаются. Галактики, например. А кошка своих размеров не поменяет, разве что какие-нибудь константы фундаментальных взаимодействий изменятся.
>>112758
Соотношение неопределённости в его обычном виде говорит про импульсы и координаты, а про момент импульса ничего не говорит. Так что такое вполне может быть.
>> No.112779 Reply
>>112758
Прочитай повнимательнее определения объектов, входящих в соотношение Гейзенберга, и определение орбитального квантового числа.
>> No.112795 Reply
>>112777
>>112779
> Соотношение неопределённости в его обычном виде говорит про импульсы и координаты
Um... Вообще-то про любые не коммутирующие операторы.
>> No.112799 Reply
>>112795
Тогда просто про определение ОКВ. Ты получил s-обиталь.
другой анон
>> No.112818 Reply
>>112799
Ну да, это я понимаю. Но я не понимаю, как такое вообще возможно. На s-орбитали определены все проекции момента импульса! Это же невозможно по идее. При этом упоминаний не нашёл ни в Ландавшице, ни в Ципенюке, ни в Сивухине, ни в Байкове-Кузнецове, сижу, обложившись книгами, и плачу. WTF. Пойду завтра лектора пытать.
>> No.112822 Reply
>>112818
А что странного? Ну нет у него этого момента. Все операторы коммутируют с нулём, если хочешь. Хотя я конечно, голос дивана. Подходи к лектору, всё-равно, это полезно.
>> No.112824 Reply
>>112795
Ну да. У тебя x и p коммутируют на i \hbar, отсюда вытекает соотношение неопределённости. А два орбитальных момента коммутируют на третий. Если первый из них - ноль, второй - ноль, и третий тоже, то какие вообще проблемы? Никаких проблем. Просто x и p образуют полный набор операторов, а моменты не образуют.
>> No.112825 Reply
>>112824
> x и p образуют полный набор операторов,
Полный набор состоит из одновременно измеримых величин, как минимум!

Но вообще вроде логично (первая часть). Спасибо, попробую разобраться.
То есть Lx и Ly коммутируют на Lz, и если все равны нулю, то... Да, получается. Анон, ты няшка!
>> No.112834 Reply
File: Untitled.png
Png, 2.48 KB, 112×263
edit Find source with google Find source with iqdb
Untitled.png
File: 350x700px-LL-7ae3...
Jpeg, 45.76 KB, 350×350
edit Find source with google Find source with iqdb
350x700px-LL-7ae38429_vc02_1.jpeg

Не нашел кафедры урока труда.

Подскажи пожалуйста как грамотно склеить конструкцию пикрелейтеда?
Черное - жесткий пластик, Красное - медный контакт.

Думаю акриловым клеем склеить, но не хочу запороть.
>> No.112841 Reply
>>112834
> Не нашел кафедры урока труда.
Так создай. Зачем это в нашем храме науки?
>>112825
> Полный набор состоит из одновременно измеримых величин, как минимум!
А, да, соврал. Простите.
>> No.113362 Reply
File: lineforce.gif
Gif, 262.39 KB, 800×372 - Click the image to expand
edit Find source with google Find source with iqdb
lineforce.gif
Няши, я к вам влезу со своим вопросом. Поясните за силовые линии. Определение я знаю, но...
Железные опилки выстраиваются вдоль силовых линий магнита. То есть поле вдоль силовых линий чем-то отличается от поля между ними, но я не помню, чтобы напряжённость и индукция убывали скачками. То есть почему они выстраиваются в линии, а не просто ориентируются вдоль вектора силы? Или всё это лишь моя ошибка, они действительно выстраиваются только вдоль вектора, а видимые линии результат других процессов?
>> No.113365 Reply
>>113362
Они выстраиваются только вдоль вектора, да. Но они еще и притягиваются друг к другу, и засчет притяжения слипаются в линии, так как их недостаточно чтобы покрыть всю поверхность. А расположение получившихся линий случайно.
>> No.113367 Reply
>>113365
Ещё добавлю, опилки в магнитном поле намагничиваются, а два ориентированных одинаково магнита отталкиваются. Так что даже если насыпать много опилок, они скорее всего образуют такую картину.
>> No.113369 Reply
File: 1318876019854.jpg
Jpg, 40.01 KB, 521×488 - Click the image to expand
edit Find source with google Find source with iqdb
1318876019854.jpg
>>113365
>>113367
> засчет притяжения слипаются в линии
> опилки в магнитном поле намагничиваются, а два ориентированных одинаково магнита отталкиваются
I have a question.
>> No.113374 Reply
>>113369
Магниты когда-нибудь в руках держал? Они притягиваются разноименными полюсами и отталкиваются одноименными. Поэтому две опилки, расположенные одна за другой, слипнутся в линию, а лежащие на одном уровне - наоборот, оттолкнутся, потому что у них одноименные концы рядом.
>> No.113375 Reply
File: lineforce.gif
Gif, 154.47 KB, 800×372 - Click the image to expand
edit Find source with google Find source with iqdb
lineforce.gif
>>113374
Ага, как охуенно они выстроились друг за другом. Было бы как ты говоришь они бы распределялись линиями толщиной в одну опилку и равномерно, а не вот так.
>> No.113376 Reply
>>113375
Да и твой принцип нельзя применить к заряженным частицам солнечного ветра, которые образуют полярные сияния.
>> No.113377 Reply
>>113375
Индуцированные магнитные поля достаточно малы, чтобы частицы могли действовать друг на друга.
Выстраиваются строго по линиям полей, а "слипаются" банально случайно из - за предварительного равномерного распределения по поверхности.
предположение уровня дивана
>> No.113378 Reply
>>113377
Правдоподобно. Но полярные сияния.
>> No.113380 Reply
>>113378
А при чём тут они?
Магнитное поле Земли действует на солнечный ветер, частицы которого закручиваются вокруг полюсов и ионизируют атмосферу.
>> No.113382 Reply
>>113377
Ой, что же я написал, еретик.
Пойду просплюсь.
>> No.113383 Reply
File: auroracircle2_curtis_big.jpg
Jpg, 34.22 KB, 400×593 - Click the image to expand
edit Find source with google Find source with iqdb
auroracircle2_curtis_big.jpg
>>113380
Так атмосфера не просто светиться, а вдоль силовых линий.
>> No.113396 Reply
>>113383
Никаких "силовых линий" именно как отдельных линий нет. В таких терминах детям объясняют, что такое векторное поле.
>> No.113399 Reply
>>113396
=>
>>113396
>>113375
Картинки смотри
>> No.113469 Reply
Годный ли учебник Мякишева для школьника?
>> No.113535 Reply
Анон, помоги заочнику с задачей.
> В центр деревянного шара радиусом 7 см, лежащего на столе, попадает пуля массой 10 г, летящая горизонтально со скоростью 350 м/c, и застревает в нём. Найти массу шара, если он после удара покатится без скольжения с угловой скоростью 22 рад/с.
Казалось бы, всё элементарно: абсолютно неупругий удар, m1v1 + m2v2 = (m1 + M2)U, скорость катящегося шара — угловая скорость на радиус. Но преподаватель почему-то подчеркнул формулу m1v1 + m2v2 = (m1 + M2)U и рядом написал: «Нельзя пользоваться этим законом!». Что я сделал не так?
>> No.113558 Reply
Анон, помоги заочнику с задачей. Мне кажется, или в условии чего-то не хватат?
> Определить, на сколько изменится длина волны де Бройля электрона, вырванного квантом с энергией 14,5 эВ с первой боровской орбиты атома водорода на большое расстояние.
>> No.113571 Reply
>>113535
Само собой, нужно использовать не закон сохранения импульса, а закон сохранения момента импульса. Разные точки шара будут двигаться с разной скоростью, нет никакого U. Момент инерции шара можно взять из таблицы и использовать теорему Гюйгенса-Штейнера. Алсо, пуля застрянет в центре шара, а не растворится по всему его объёму, хотя, возможно, её массой в шаре лучше просто пренебречь.
>> No.113577 Reply
диффундируем в >113576
>> No.113578 Reply


Password:

[ /tv/ /rf/ /vg/ /a/ /b/ /u/ /bo/ /fur/ /to/ /dt/ /cp/ /oe/ /bg/ /ve/ /r/ /mad/ /d/ /mu/ /cr/ /di/ /sw/ /hr/ /wh/ /lor/ /s/ /hau/ /slow/ /gf/ /vn/ /w/ /ma/ /azu/ /wn/ ] [ Main | Settings | Bookmarks | Music Player ]